You are on page 1of 161

Digestive System Disorders

1. A patient with chronic alcohol abuse is admitted with liver failure.


You closely monitor the patients blood pressure because of which
change that is associated with the liver failure?

1. Hypoalbuminemia
2. Increased capillary permeability
3. Abnormal peripheral vasodilation
4. Excess rennin release from the kidneys

2. Youre assessing the stoma of a patient with a healthy, well-healed


colostomy. You expect the stoma to appear:

1. Pale, pink and moist


2. Red and moist
3. Dark or purple colored
4. Dry and black

3. Youre caring for a patient with a sigmoid colostomy. The stool from
this colostomy is:

1. Formed
2. Semisolid
3. Semiliquid
4. Watery

4. Youre advising a 21 y.o. with a colostomy who reports problems


with flatus. What food should you recommend?

1. Peas
2. Cabbage
3. Broccoli
4. Yogurt

5. You have to teach ostomy self care to a patient with a colostomy.


You tell the patient to measure and cut the wafer:

1. To the exact size of the stoma.


2. About 1/16 larger than the stoma.
3. About 1/8 larger than the stoma.
4. About 1/4 larger than the stoma.

6. Youre performing an abdominal assessment on Brent who is 52 y.o.


In which order do you proceed?

1. Observation, percussion, palpation, auscultation


2. Observation, auscultation, percussion, palpation
3. Percussion, palpation, auscultation, observation
4. Palpation, percussion, observation, auscultation

7. Youre doing preoperative teaching with Gertrude who has ulcerative


colitis who needs surgery to create an ileoanal reservoir. Which
information do you include?

1. A reservoir is created that exits through the abdominal wall.


2. A second surgery is required 12 months after the first surgery.
3. A permanent ileostomy is created.
4. The surgery occurs in two stages.

8. Youre caring for Carin who has just had ileostomy surgery. During
the first 24 hours post-op, how much drainage can you expect from the
ileostomy?
1. 100 ml
2. 500 ml
3. 1500 ml
4. 5000 ml

9. Youre preparing a teaching plan for a 27 y.o. named Jeff who


underwent surgery to close a temporary ileostomy. Which nutritional
guideline do you include in this plan?

1. There is no need to change eating habits.


2. Eat six small meals a day.
3. Eat the largest meal in the evening.
4. Restrict fluid intake.

10. Arthur has a family history of colon cancer and is scheduled to have
a sigmoidoscopy. He is crying as he tells you, I know that I have
colon cancer, too. Which response is most therapeutic?

1. I know just how you feel.


2. You seem upset.
3. Oh, dont worry about it, everything will be just fine.
4. Why do you think you have cancer?

11. Youre caring for Beth who underwent a Billroth II procedure


(surgical removal of the pylorus and duodenum) for treatment of a
peptic ulcer. Which findings suggest that the patient is developing
dumping syndrome, a complication associated with this procedure?

1. Flushed, dry skin.


2. Headache and bradycardia.
3. Dizziness and sweating.
4. Dyspnea and chest pain.
12. Youre developing the plan of care for a patient experiencing
dumping syndrome after a Billroth II procedure. Which dietary
instructions do you include?

1. Omit fluids with meals.


2. Increase carbohydrate intake.
3. Decrease protein intake.
4. Decrease fat intake.

13. Youre caring for Lewis, a 67 y.o. patient with liver cirrhosis who
developed ascites and requires paracentesis. Relief of which symptom
indicated that the paracentesis was effective?

1. Pruritus
2. Dyspnea
3. Jaundice
4. Peripheral Neuropathy

14. Youre caring for Jane, a 57 y.o. patient with liver cirrhosis who
developed ascites and requires paracentesis. Before her paracentesis,
you instruct her to:

1. Empty her bladder.


2. Lie supine in bed.
3. Remain NPO for 4 hours.
4. Clean her bowels with an enema.

15. After abdominal surgery, your patient has a severe coughing


episode that causes wound evisceration. In addition to calling the
doctor, which intervention is most appropriate?

1. Irrigate the wound & organs with Betadine.


2. Cover the wound with a saline soaked sterile dressing.
3. Apply a dry sterile dressing & binder.
4. Push the organs back & cover with moist sterile dressings.

16. Youre caring for Betty with liver cirrhosis. Which of the following
assessment findings leads you to suspect hepatic encephalopathy in
her?

1. Asterixis
2. Chvosteks sign
3. Trousseaus sign
4. Hepatojugular reflex

17. You are developing a care plan on Sally, a 67 y.o. patient with
hepatic encephalopathy. Which of the following do you include?

1. Administering a lactulose enema as ordered.


2. Encouraging a protein-rich diet.
3. Administering sedatives, as necessary.
4. Encouraging ambulation at least four times a day.

18. You have a patient with achalasia (incomplete muscle relaxtion of


the GI tract, especially sphincter muscles). Which medications do you
anticipate to administer?

1. Isosorbide dinitrate (Isordil)


2. Digoxin (Lanoxin)
3. Captopril (Capoten)
4. Propranolol (Inderal)

19. The student nurse is preparing a teaching care plan to help improve
nutrition in a patient with achalasia. You include which of the following:
1. Swallow foods while leaning forward.
2. Omit fluids at mealtimes.
3. Eat meals sitting upright.
4. Avoid soft and semi soft foods.

20. Britney, a 20 y.o. student is admitted with acute pancreatitis. Which


laboratory findings do you expect to be abnormal for this patient?

1. Serum creatinine and BUN


2. Alanine aminotransferase (ALT) and aspartate aminotransferase (AST)
3. Serum amylase and lipase
4. Cardiac enzymes

21. A patient with Crohns disease is admitted after 4 days of diarrhea.


Which of the following urine specific gravity values do you expect to
find in this patient?

1. 1.005
2. 1.011
3. 1.020
4. 1.030

22. Your goal is to minimize Davids risk of complications after a


herniorrhaphy. You instruct the patient to:

1. Avoid the use of pain medication.


2. Cough and deep breathe Q2H.
3. Splint the incision if he cant avoid sneezing or coughing.
4. Apply heat to scrotal swelling.

23. Janice is waiting for discharge instructions after her herniorrhaphy.


Which of the following instructions do you include?
1. Eat a low-fiber diet.
2. Resume heavy lifting in 2 weeks.
3. Lose weight, if obese.
4. Resume sexual activity once discomfort is gone.

24. Develop a teaching care plan for Angie who is about to undergo a
liver biopsy. Which of the following points do you include?

1. Youll need to lie on your stomach during the test.


2. Youll need to lie on your right side after the test.
3. During the biopsy youll be asked to exhale deeply and hold it.
4. The biopsy is performed under general anesthesia.

25. Stephen is a 62 y.o. patient that has had a liver biopsy. Which of the
following groups of signs alert you to a possible pneumothorax?

1. Dyspnea and reduced or absent breath sounds over the right lung
2. Tachycardia, hypotension, and cool, clammy skin
3. Fever, rebound tenderness, and abdominal rigidity
4. Redness, warmth, and drainage at the biopsy site

26. Michael, a 42 y.o. man is admitted to the med-surg floor with a


diagnosis of acute pancreatitis. His BP is 136/76, pulse 96, Resps 22
and temp 101. His past history includes hyperlipidemia and alcohol
abuse. The doctor prescribes an NG tube. Before inserting the tube, you
explain the purpose to patient. Which of the following is a most
accurate explanation?

1. It empties the stomach of fluids and gas.


2. It prevents spasms at the sphincter of Oddi.
3. It prevents air from forming in the small intestine and large intestine.
4. It removes bile from the gallbladder.
27. Jason, a 22 y.o. accident victim, requires an NG tube for feeding.
What should you immediately do after inserting an NG tube for liquid
enteral feedings?

1. Aspirate for gastric secretions with a syringe.


2. Begin feeding slowly to prevent cramping.
3. Get an X-ray of the tip of the tube within 24 hours.
4. Clamp off the tube until the feedings begin.

28. Stephanie, a 28 y.o. accident victim, requires TPN. The rationale for
TPN is to provide:

1. Necessary fluids and electrolytes to the body.


2. Complete nutrition by the I.V. route.
3. Tube feedings for nutritional supplementation.
4. Dietary supplementation with liquid protein given between meals.

29. Type A chronic gastritis can be distinguished from type B by its


ability to:

1. Cause atrophy of the parietal cells.


2. Affect only the antrum of the stomach.
3. Thin the lining of the stomach walls.
4. Decrease gastric secretions.

30. Matt is a 49 y.o. with a hiatal hernia that you are about to counsel.
Health care counseling for Matt should include which of the following
instructions?

1. Restrict intake of high-carbohydrate foods.


2. Increase fluid intake with meals.
3. Increase fat intake.
4. Eat three regular meals a day.
31. Jerod is experiencing an acute episode of ulcerative colitis. Which is
priority for this patient?

1. Replace lost fluid and sodium.


2. Monitor for increased serum glucose level from steroid therapy.
3. Restrict the dietary intake of foods high in potassium.
4. Note any change in the color and consistency of stools.

32. A 29 y.o. patient has an acute episode of ulcerative colitis. What


diagnostic test confirms this diagnosis?

1. Barium Swallow.
2. Stool examination.
3. Gastric analysis.
4. Sigmoidoscopy.

33. Eleanor, a 62 y.o. woman with diverticulosis is your patient. Which


interventions would you expect to include in her care?

1. Low-fiber diet and fluid restrictions.


2. Total parenteral nutrition and bed rest.
3. High-fiber diet and administration of psyllium.
4. Administration of analgesics and antacids.

34. Regina is a 46 y.o. woman with ulcerative colitis. You expect her
stools to look like:

1. Watery and frothy.


2. Bloody and mucous.
3. Firm and well-formed.
4. Alternating constipation and diarrhea.
35. Donald is a 61 y.o. man with diverticulitis. Diverticulitis is
characterized by:

1. Periodic rectal hemorrhage.


2. Hypertension and tachycardia.
3. Vomiting and elevated temperature.
4. Crampy and lower left quadrant pain and low-grade fever.

36. Brenda, a 36 y.o. patient is on your floor with acute pancreatitis.


Treatment for her includes:

1. Continuous peritoneal lavage.


2. Regular diet with increased fat.
3. Nutritional support with TPN.
4. Insertion of a T tube to drain the pancreas.

37. Glenda has cholelithiasis (gallstones). You expect her to complain


of:

1. Pain in the right upper quadrant, radiating to the shoulder.


2. Pain in the right lower quadrant, with rebound tenderness.
3. Pain in the left upper quadrant, with shortness of breath.
4. Pain in the left lower quadrant, with mild cramping.

38. After an abdominal resection for colon cancer, Madeline returns to


her room with a Jackson-Pratt drain in place. The purpose of the drain
is to:

1. Irrigate the incision with a saline solution.


2. Prevent bacterial infection of the incision.
3. Measure the amount of fluid lost after surgery.
4. Prevent accumulation of drainage in the wound.
39. Anthony, a 60 y.o. patient, has just undergone a bowel resection
with a colostomy. During the first 24 hours, which of the following
observations about the stoma should you report to the doctor?

1. Pink color.
2. Light edema.
3. Small amount of oozing.
4. Trickles of bright red blood.

40. Your teaching Anthony how to use his new colostomy. How much
skin should remain exposed between the stoma and the ring of the
appliance?

1. 1/16
2. 1/4
3. 1/2
4. 1

41. Claire, a 33 y.o. is on your floor with a possible bowel obstruction.


Which intervention is priority for her?

1. Obtain daily weights.


2. Measure abdominal girth.
3. Keep strict intake and output.
4. Encourage her to increase fluids.

42. Your patient has a GI tract that is functioning, but has the inability
to swallow foods. Which is the preferred method of feeding for your
patient?

1. TPN
2. PPN
3. NG feeding
4. Oral liquid supplements

43. Youre patient is complaining of abdominal pain during assessment.


What is your priority?

1. Auscultate to determine changes in bowel sounds.


2. Observe the contour of the abdomen.
3. Palpate the abdomen for a mass.
4. Percuss the abdomen to determine if fluid is present.

44. Before bowel surgery, Lee is to administer enemas until clear.


During administration, he complains of intestinal cramps. What do you
do next?

1. Discontinue the procedure.


2. Lower the height of the enema container.
3. Complete the procedure as quickly as possible.
4. Continue administration of the enema as ordered without making any
adjustments.

45. Leigh Ann is receiving pancrelipase (Viokase) for


chronic pancreatitis. Which observation best indicates the treatment is
effective?

1. There is no skin breakdown.


2. Her appetite improves.
3. She loses more than 10 lbs.
4. Stools are less fatty and decreased in frequency.

46. Ralph has a history of alcohol abuse and has acute pancreatitis.
Which lab value is most likely to be elevated?
1. Calcium
2. Glucose
3. Magnesium
4. Potassium

47. Anna is 45 y.o. and has a bleeding ulcer. Despite multiple blood
transfusions, her HGB is 7.5g/dl and HCT is 27%. Her doctor
determines that surgical intervention is necessary and she undergoes
partial gastrectomy. Postoperative nursing care includes:

1. Giving pain medication Q6H.


2. Flushing the NG tube with sterile water.
3. Positioning her in high Fowlers position.
4. Keeping her NPO until the return of peristalsis.

48. Sitty, a 66 y.o. patient underwent a colostomy for ruptured


diverticulum. She did well during the surgery and returned to your
med-surg floor in stable condition. You assess her colostomy 2 days
after surgery. Which finding do you report to the doctor?

1. Blanched stoma
2. Edematous stoma
3. Reddish-pink stoma
4. Brownish-black stoma

49. Sharon has cirrhosis of the liver and develops ascites. What
intervention is necessary to decrease the excessive accumulation of
serous fluid in her peritoneal cavity?

1. Restrict fluids
2. Encourage ambulation
3. Increase sodium in the diet
4. Give antacids as prescribed
50. Katrina is diagnosed with lactose intolerance. To avoid
complications with lack of calcium in the diet, which food should be
included in the diet?

1. Fruit
2. Whole grains
3. Milk and cheese products
4. Dark green, leafy vegetables

51. Nathaniel has severe pruritus due to having hepatitis B. What is the
best intervention for his comfort?

1. Give tepid baths.


2. Avoid lotions and creams.
3. Use hot water to increase vasodilation.
4. Use cold water to decrease the itching.

52. Rob is a 46 y.o. admitted to the hospital with a suspected diagnosis


of Hepatitis B. Hes jaundiced and reports weakness. Which
intervention will you include in his care?

1. Regular exercise.
2. A low-protein diet.
3. Allow patient to select his meals.
4. Rest period after small, frequent meals.

53. Youre discharging Nathaniel with hepatitis B. Which statement


suggests understanding by the patient?

1. Now I can never get hepatitis again.


2. I can safely give blood after 3 months.
3. Ill never have a problem with my liver again, even if I drink alcohol.
4. My family knows that if I get tired and start vomiting, I may be getting sick
again.

54. Gail is scheduled for a cholecystectomy. After completion of


preoperative teaching, Gail states,If I lie still and avoid turning after
the operation, Ill avoid pain. Do you think this is a good idea? What is
the best response?

1. Youll need to turn from side to side every 2 hours.


2. Its always a good idea to rest quietly after surgery.
3. The doctor will probably order you to lie flat for 24 hours.
4. Why dont you decide about activity after you return from the recovery
room?

55. Youre caring for a 28 y.o. woman with hepatitis B. Shes concerned
about the duration of her recovery. Which response isnt appropriate?

1. Encourage her to not worry about the future.


2. Encourage her to express her feelings about the illness.
3. Discuss the effects of hepatitis B on future health problems.
4. Provide avenues for financial counseling if she expresses the need.

56. Elmer is scheduled for a proctoscopy and has an I.V. The doctor
wrote an order for 5mg of I.V. diazepam(Valium). Which order is
correct regarding diazepam?

1. Give diazepam in the I.V. port closest to the vein.


2. Mix diazepam with 50 ml of dextrose 5% in water and give over 15 minutes.
3. Give diazepam rapidly I.V. to prevent the bloodstream from diluting the drug
mixture.
4. Question the order because I.V. administration of diazepam is
contraindicated.
57. Annabelle is being discharged with a colostomy, and youre
teaching her about colostomy care. Which statement correctly
describes a healthy stoma?

1. At first, the stoma may bleed slightly when touched.


2. The stoma should appear dark and have a bluish hue.
3. A burning sensation under the stoma faceplate is normal.
4. The stoma should remain swollen away from the abdomen.

58. A patient who underwent abdominal surgery now has a gaping


incision due to delayed wound healing. Which method is correct when
you irrigate a gaping abdominal incision with sterile normal
saline solution, using a piston syringe?

1. Rapidly instill a stream of irrigating solution into the wound.


2. Apply a wet-to-dry dressing to the wound after the irrigation.
3. Moisten the area around the wound with normal saline solution after the
irrigation.
4. Irrigate continuously until the solution becomes clear or all of the solution is
used.

59. Hepatic encephalopathy develops when the blood level of which


substance increases?

1. Ammonia
2. Amylase
3. Calcium
4. Potassium

60. Your patient recently had abdominal surgery and tells you that he
feels a popping sensation in his incision during a coughing spell,
followed by severe pain. You anticipate an evisceration. Which supplies
should you take to his room?
1. A suture kit.
2. Sterile water and a suture kit.
3. Sterile water and sterile dressings.
4. Sterile saline solution and sterile dressings.

61. Findings during an endoscopic exam include a cobblestone


appearance of the colon in your patient. The findings are characteristic
of which disorder?

1. Ulcer
2. Crohns disease
3. Chronic gastritis
4. Ulcerative colitis

62. What information is correct about stomach cancer?

1. Stomach pain is often a late symptom.


2. Surgery is often a successful treatment.
3. Chemotherapy and radiation are often successful treatments.
4. The patient can survive for an extended time with TPN.

63. Dark, tarry stools indicate bleeding in which location of the GI


tract?

1. Upper colon.
2. Lower colon.
3. Upper GI tract.
4. Small intestine.

64. A patient has an acute upper GI hemorrhage. Your interventions


include:
1. Treating hypovolemia.
2. Treating hypervolemia.
3. Controlling the bleeding source.
4. Treating shock and diagnosing the bleeding source.

65. You promote hemodynamic stability in a patient with upper


GI bleeding by:

1. Encouraging oral fluid intake.


2. Monitoring central venous pressure.
3. Monitoring laboratory test results and vital signs.
4. Giving blood, electrolyte and fluid replacement.

66. Youre preparing a patient with a malignant tumor for colorectal


surgery and subsequent colostomy. The patient tells you hes anxious.
What should your initial step be in working with this patient?

1. Determine what the patient already knows about colostomies.


2. Show the patient some pictures of colostomies.
3. Arrange for someone who has a colostomy to visit the patient.
4. Provide the patient with written material about colostomy care.

67. Your patient, Christopher, has a diagnosis of ulcerative colitis and


has severe abdominal pain aggravated by movement, rebound
tenderness, fever, nausea, and decreased urine output. This may
indicate which complication?

1. Fistula.
2. Bowel perforation.
3. Bowel obstruction.
4. Abscess.
68. A patient has a severe exacerbation of ulcerative colitis. Long-term
medications will probably include:

1. Antacids.
2. Antibiotics.
3. Corticosteroids.
4. Histamine2-receptor blockers.

69. The student nurse is teaching the family of a patient with liver
failure. You instruct them to limit which foods in the patients diet?

1. Meats and beans.


2. Butter and gravies.
3. Potatoes and pastas.
4. Cakes and pastries.

70. An intubated patient is receiving continuous enteral feedings


through a Salem sump tube at a rate of 60ml/hr. Gastric residuals have
been 30-40ml when monitored Q4H. You check the gastric residual and
aspirate 220ml. What is your first response to this finding?

1. Notify the doctor immediately.


2. Stop the feeding, and clamp the NG tube.
3. Discard the 220ml, and clamp the NG tube.
4. Give a prescribed GI stimulant such as metoclopramide (Reglan).

71. Your patient with peritonitis is NPO and complaining of thirst. What
is your priority?

1. Increase the I.V. infusion rate.


2. Use diversion activities.
3. Provide frequent mouth care.
4. Give ice chips every 15 minutes.
72. Kevin has a history of peptic ulcer disease and vomits coffee-ground
emesis. What does this indicate?

1. He has fresh, active upper GI bleeding.


2. He needs immediate saline gastric lavage.
3. His gastric bleeding occurred 2 hours earlier.
4. He needs a transfusion of packed RBCs.

73. A 53 y.o. patient has undergone a partial gastrectomy for


adenocarcinoma of the stomach. An NG tube is in place and is
connected to low continuous suction. During the immediate
postoperative period, you expect the gastric secretions to be which
color?

1. Brown.
2. Clear.
3. Red.
4. Yellow.

74. Your patient has a retractable gastric peptic ulcer and has had a
gastric vagotomy. Which factor increases as a result of vagotomy?

1. Peristalsis.
2. Gastric acidity.
3. Gastric motility.
4. Gastric pH.

75. Christina is receiving an enteral feeding that requires a


concentration of 80 ml of supplement mixed with 20 ml of water. How
much water do you mix with an 8 oz (240ml) can of feeding?

1. 60 ml.
2. 70 ml.
3. 80 ml.
4. 90 ml.

76. Which stoma would you expect a malodorous, enzyme-rich, caustic


liquid output that is yellow, green, or brown?

1. Ileostomy.
2. Ascending colostomy.
3. Transverse colostomy.
4. Descending colostomy.

77. George has a T tube in place after gallbladder surgery. Before


discharge, what information or instructions should be given regarding
the T tube drainage?

1. If there is any drainage, notify the surgeon immediately.


2. The drainage will decrease daily until the bile duct heals.
3. First, the drainage is dark green; then it becomes dark yellow.
4. If the drainage stops, milk the tube toward the puncture wound.

78. Your patient Maria takes NSAIDS for her degenerative joint disease,
has developed peptic ulcer disease. Which drug is useful in preventing
NSAID-induced peptic ulcer disease?

1. Calcium carbonate (Tums)


2. Famotidine (Pepcid)
3. Misoprostol (Cytotec)
4. Sucralfate (Carafate)

79. The student nurse is participating in colorectal cancer-screening


program. Which patient has the fewest risk factors for colon cancer?
1. Janice, a 45 y.o. with a 25-year history of ulcerative colitis
2. George, a 50 y.o. whose father died of colon cancer
3. Herman, a 60 y.o. who follows a low-fat, high-fiber diet
4. Sissy, a 72 y.o. with a history of breast cancer

80. Youre patient, post-op drainage of a pelvic abscess secondary to


diverticulitis, begins to cough violently after drinking water. His wound
has ruptured and a small segment of the bowel is protruding. Whats
your priority?

1. Ask the patient what happened, call the doctor, and cover the area with a
water-soaked bedsheet.
2. Obtain vital signs, call the doctor, and obtain emergency orders.
3. Have a CAN hold the wound together while you obtain vital signs, call the
doctor and flex the patients knees.
4. Have the doctor called while you remain with the patient, flex the patients
knees, and cover the wound with sterile towels soaked in sterile saline solution.

Answers and Rationale

1. Answer: 1. Hypoalbuminemia

Blood pressure decreases as the body is unable to maintain normal oncotic


pressure with liver failure, so patients with liver failure require close blood
pressure monitoring. Increased capillary permeability, abnormal peripheral
vasodilation, and excess rennin released from the kidneys arent direct
ramifications of liver failure.

2. Answer: 2. Red and moist

Good circulation causes tissues to be moist and red, so a healthy, well-healed


stoma appears red and moist.
3. Answer: 1. Formed

A colostomy in the sigmoid colon produces a solid, formed stool.

4. Answer: 4. Yogurt

High-fiber foods stimulate peristalsis, and a result, flatus. Yogurt reduces gas
formation.

5. Answer: 2. About 1/16 larger than the stoma.

A proper fit protects the skin but doesnt impair circulation. A 1/16 should be
cut.

6. Answer: 2. Observation, auscultation, percussion, palpation

Observation, auscultation, percussion, palpation

7. Answer: 4. The surgery occurs in two stages.

An ileoanal reservoir is created in two stages. The two surgeries are about 2 to
3 months apart. First, diseased intestines are removed and a temporary loop
ileostomy is created. Second, the loop ileostomy is closed and stool goes to the
reservoir and out through the anus.

8. Answer: 3. 1500 ml

The large intestine absorbs large amounts of water so the initial output from the
ileostomy may be as much as 1500 to 2000 ml/24 hours. Gradually, the small
intestine absorbs more fluid and the output decreases.

9. Answer: 2. Eat six small meals a day


To avoid overloading the small intestine, encourage the patient to eat six small,
regularly spaced meals.

10. Answer: 2. You seem upset.

Making observations about what you see or hear is a useful therapeutic


technique. This way, you acknowledge that you are interested in what the
patient is saying and feeling.

11. Answer: 3. Dizziness and sweating.

After a Billroth II procedure, a large amount of hypertonic fluid enters the


intestine. This causes extracellular fluid to move rapidly into the bowel, reducing
circulating blood volume and producing vasomotor symptoms. Vasomotor
symptoms produced by dumping syndrome include dizziness and sweating,
tachycardia, syncope, pallor, and palpitations.

12. Answer: 1. Omit fluids with meals.

Gastric emptying time can be delayed by omitting fluids from your patients
meal. A diet low in carbs and high in fat & protein is recommended to treat
dumping syndrome.

13. Answer: 2. Dyspnea

Ascites puts pressure on the diaphragm. Paracentesis is done to remove fluid


and reducing pressure on the diaphragm. The goal is to improve the patients
breathing. The others are signs of cirrhosis that arent relieved by paracentesis.

14. Answer: 1. Empty her bladder.

A full bladder can interfere with paracentesis and be punctured inadvertently.


15. Answer: 2. Cover the wound with a saline-soaked sterile dressing.

Cover the organs with a sterile, nonadherent dressing moistened with normal
saline. Do this to prevent infection and to keep the organs from drying out.

16. Answer: 1. Asterixis

Asterixis is an early neurologic sign of hepatic encephalopathy elicited by asking


the patient to hold her arms stretched out. Asterixis is present if the hands
rapidly extend and flex.

17. Answer: 1. Administering a lactulose enema as ordered.

You may administer the laxative lactulose to reduce ammonia levels in the
colon.

18. Answer: 1. Isosorbide dinitrate (Isordil)

Achalasia is characterized by incomplete relaxation of the LES, dilation of the


lower esophagus, and a lack of esophageal peristalsis. Because nitrates relax
the lower esophageal sphincter, expect to give Isordil orally or sublingually.

19. Answer: 3. Eat meals sitting upright.

Eating in the upright position aids in emptying the esophagus. Doing the
opposite of the other three also may be helpful.

20. Answer: 3. Serum amylase and lipase

Pancreatitis involves activation of pancreatic enzymes, such as amylase and


lipase. These levels are elevated in a patient with acute pancreatitis.

21. Answer: 4. 1.030


The normal range of specific gravity of urine is 1.010 to 1.025; a value of 1.030
may be seen with dehydration.

22. Answer: 3. Splint the incision if he cant avoid sneezing or coughing.

Teach the pt to avoid activities that increase intra-abdominal pressure such as


coughing, sneezing, or straining with a bowel movement.

23. Answer: 3. Lose weight, if obese.

Because obesity weakens the abdominal muscles, advise weight loss for the
patient who has had a hernia repair.

24. Answer: 2. Youll need to lie on your right side after the test.

After a liver biopsy, the patient is placed on the right side to compress the liver
and to reduce the risk of bleeding or bile leakage.

25. Answer: 1. Dyspnea and reduced or absent breath sounds over the
right lung

Signs and Symptoms of pneumothorax include dyspnea and decreased or


absent breath sounds over the affected lung (right lung).

26. Answer: 1. It empties the stomach of fluids and gas.

An NG tube is inserted into the patients stomach to drain fluid and gas.

27. Answer: 1. Aspirate for gastric secretions with a syringe.

Aspirating the stomach contents confirms correct placement. If an X-ray is


ordered, it should be done immediately, not in 24 hours.
28. Answer: 2. Complete nutrition by the I.V. route.

TPN is given I.V. to provide all the nutrients your patient needs. TPN isnt a tube
feeding nor is it a liquid dietary supplement.

29. Answer: 1. Cause atrophy of the parietal cells.

Type A causes changes in parietal cells.

30. Answer: 2. Increase fluid intake with meals.

Increasing fluids helps empty the stomach. A high carb diet isnt restricted and
fat intake shouldnt be increased.

31. Answer: 1. Replace lost fluid and sodium.

Diarrhea d/t an acute episode of ulcerative colitis leads to fluid & electrolyte
losses so fluid replacement takes priority.

32. Answer: 4. Sigmoidoscopy.

Sigmoidoscopy allows direct observation of the colon mucosa for changes, and if
needed, biopsy.

33. Answer: 3. High-fiber diet and administration of psyllium.

She needs a high-fiber diet and a psyllium (bulk laxative) to promote normal
soft stools.

34. Answer: 2. Bloody and mucous.

Stools from ulcerative colitis are often bloody and contain mucus.
35. Answer: 4. Crampy and lower left quadrant pain and low-
grade fever.

One sign of acute diverticulitis is crampy lower left quadrant pain. A low-
grade fever is another common sign.

36. Answer: 3. Nutritional support with TPN.

With acute pancreatitis, you need to rest the GI tract by TPN as nutritional
support.

37. Answer: 1. Pain in the right upper quadrant, radiating to the


shoulder.

The gallbladder is located in the RUQ and a frequent sign of gallstones is pain
radiating to the shoulder.

38. Answer: 4. Prevent accumulation of drainage in the wound.

A Jackson-Pratt drain promotes wound healing by allowing fluid to escape from


the wound.

39. Answer: 4. Trickles of bright red blood.

After creation of a colostomy, expect to see a stoma that is pink, slightly


edematous, with some oozing. Bright red blood, regardless of amount,
indicates bleeding and should be reported to the doctor.

40. Answer: 1. 1/16

Only a small amount of skin should be exposed and more than 1/16 of skin
allows the excrement to irritate the skin.
41. Answer: 2. Measure abdominal girth.

Measuring abdominal girth provides quantitative information about increases or


decreases in the amount of distention.

42. Answer: 3. NG feeding

Because the GI tract is functioning, feeding methods involve the enteral route
which bypasses the mouth but allows for a major portion of the GI tract to be
used.

43. Answer: 2. Observe the contour of the abdomen.

The first step in assessing the abdomen is to observe its shape and contour,
then auscultate, palpate, and then percuss.

44. Answer: 2. Lower the height of the enema container.

Lowering the height decreases the amount of flow, allowing him to tolerate
more fluid.

45. Answer: 4. Stools are less fatty and decreased in frequency.

Pancrelipase provides the exocrine pancreatic enzyme necessary for proper


protein, fat, and carb digestion. With increased fat digestion and absorption,
stools become less frequent and normal in appearance.

46. Answer: 2. Glucose

Glucose level increases and diabetes mellitus may result d/t the pancreatic
damage to the islets of Langerhans.

47. Answer: 4. Keeping her NPO until the return of peristalsis.


After surgery, she remains NPO until peristaltic activity returns. This decreases
the risk for abdominal distention and obstruction.

48. Answer: 4. Brownish-black stoma

A brownish-black color indicates lack of blood flow, and maybe necrosis.

49. Answer: 1. Restrict fluids

Restricting fluids decrease the amount of body fluid and the accumulation of
fluid in the peritoneal space.

50. Answer: 4. Dark green, leafy vegetables

Dark green, leafy vegetables are rich in calcium.

51. Answer: 1. Give tepid baths.

For pruritus, care should include tepid sponge baths and use of emollient
creams and lotions.

52. Answer: 4. Rest period after small, frequent meals.

Rest periods and small frequent meals are indicated during the acute phase of
hepatitis B.

53. Answer: 4. My family knows that if I get tired and start vomiting, I
may be getting sick again.

Hepatitis B can recur. Patients who have had hepatitis are permanently barred
from donating blood. Alcohol is metabolized by the liver and should be avoided
by those who have or had hepatitis B.
54. Answer: 1. Youll need to turn from side to side every 2 hours.

To prevent venous stasis and improve muscle tone, circulation, and respiratory
function, encourage her to move after surgery.

55. Answer: 1. Encourage her to not worry about the future.

Telling her not to worry minimizes her feelings.

56. Answer: 1. Give diazepam in the I.V. port closest to the vein.

Diazepam is absorbed by the plastic I.V. tubing and should be given in the port
closest to the vein.

57. Answer: 1. At first, the stoma may bleed slightly when touched.

For the first few days to a week, slight bleeding normally occurs when the
stoma is touched because the surgical site is still new. She should report
profuse bleeding immediately.

58. Answer: 4. Irrigate continuously until the solution becomes clear or


all of the solution is used.

To wash away tissue debris and drainage effectively, irrigate the wound until the
solution becomes clear or all the solution is used.

59. Answer: 1. Ammonia

Ammonia levels increase d/t improper shunting of blood, causing ammonia to


enter systemic circulation, which carries it to the brain.

60. Answer: 4. Sterile saline solution and sterile dressings.


Saline solution is isotonic, or close to body fluids in content, and is used along
with sterile dressings to cover an eviscerated wound and keep it moist.

61. Answer: 2. Crohns disease

Crohns disease penetrates the mucosa of the colon through all layers and
destroys the colon in patches, which creates a cobblestone appearance.

62. Answer: 1. Stomach pain is often a late symptom.

Stomach pain is often a late sign of stomach cancer; outcomes are particularly
poor when cancer reaches that point. Surgery, chemotherapy, and radiation
have minimal positive effects. TPN may enhance the growth of cancer.

63. Answer: 3. Upper GI tract.

Melena is the passage of dark, tarry stools that contain a large amount of
digested blood. It occurs with bleeding from the upper GI tract.

64. Answer: 1. Treating hypovolemia.

A patient with an acute upper GI hemorrhage must be treated


for hypovolemia and hemorrhagic shock. You as a nurse cant diagnose the
problem. Controlling the bleeding may require surgery or intensive medical
treatment.

65. Answer: 4. Giving blood, electrolyte and fluid replacement.

To stabilize a patient with acute bleeding, NS or LR solution is given I.V. until BP


rises and urine output returns to 30ml/hr.

66. Answer: 1. Determine what the patient already knows about


colostomies.
Initially, you should assess the patients knowledge about colostomies and how
it will affect his lifestyle.

67. Answer: 2. Bowel perforation.

An inflammatory condition that affects the surface of the colon, ulcerative colitis
causes friability and erosions with bleeding. Patients with ulcerative colitis are at
increased risk for bowel perforation, toxic megacolon, hemorrhage, cancer, and
other anorectal and systemic complications.

68. Answers: 3. Corticosteroids.

Medications to control inflammation such as corticosteroids are used for long-


term treatment.

69. Answer: 1. Meats and beans.

Meats and beans are high-protein foods. In liver failure, the liver is unable to
metabolize protein adequately, causing protein by-products to build up in the
body rather than be excreted.

70. Answer: 2. Stop the feeding, and clamp the NG tube.

A gastric residual greater than 2 hours worth of feeding or 100-150ml is


considered too high. The feeding should be stopped; NG tube clamped, and
then allow time for the stomach to empty before additional feeding is added.

71. Answer: 3. Provide frequent mouth care.

Frequent mouth care helps relieve dry mouth.

72. Answer: 3. His gastric bleeding occurred 2 hours earlier.


Coffee-ground emesis occurs when there is upper GI bleeding that has
undergone gastric digestion. For blood to appear as coffee-ground emesis, it
would have to be digested for approximately 2 hours.

73. Answer: 3. Red.

Normally, drainage is bloody for the first 24 hours after a partial gastrectomy;
then it changes to brown-tinged and then to yellow or clear.

74. Answer: 4. Gastric pH.

If the vagus nerve is cut as it enters the stomach, gastric acid secretion is
decreased, but intestinal motility is also decreased and gastric emptying is
delayed. Because gastric acids are decreased, gastric pH increases.

75. Answer: 1. 60 ml.

Dosage problem. Its 80/20 = 240/X. X=60.

76. Answer: 1. Ileostomy.

The output from an Ileostomy is described.

77. Answer: 2. The drainage will decrease daily until the bile duct
heals.

As healing occurs from the bile duct, bile drains from the tube; the amount of
bile should decrease. Teach the patient to expect dark green drainage and to
notify the doctor if drainage stops.

78. Answer: 3. Misoprostol (Cytotec)


Misoprostol restores prostaglandins that protect the stomach from NSAIDS,
which diminish the prostaglandins.

79. Answer: 3. Herman, a 60 y.o. who follows a low-fat, high-fiber diet

80. Answer: 4. Have the doctor called while you remain with the
patient, flex the patients knees, and cover the wound with sterile
towels soaked in sterile saline solution.

1. Which of the following complications is thought to be the most


common cause of appendicitis?

1. A fecalith
2. Bowel kinking
3. Internal bowel occlusion
4. Abdominal bowel swelling

2. Which of the following terms best describes the pain associated


with appendicitis?

1. Aching
2. Fleeting
3. Intermittent
4. Steady

3. Which of the following nursing interventions should be implemented


to manage a client with appendicitis?

1. Assessing for pain


2. Encouraging oral intake of clear fluids
3. Providing discharge teaching
4. Assessing for symptoms of peritonitis
4. Which of the following definitions best describes gastritis?

1. Erosion of the gastric mucosa


2. Inflammation of a diverticulum
3. Inflammation of the gastric mucosa
4. Reflux of stomach acid into the esophagus

5. Which of the following substances is most likely to cause gastritis?

1. Milk
2. Bicarbonate of soda, or baking soda
3. Enteric coated aspirin
4. Nonsteriodal anti-imflammatory drugs

6. Which of the following definitions best describes diverticulosis?

1. An inflamed outpouching of the intestine


2. A noninflamed outpouching of the intestine
3. The partial impairment of the forward flow of intestinal contents
4. An abnormal protrusion of an organ through the structure that usually holds
it.

7. Which of the following types of diets is implicated in the


development of diverticulosis?

1. Low-fiber diet
2. High-fiber diet
3. High-protein diet
4. Low-carbohydrate diet

8. Which of the following mechanisms can facilitate the development of


diverticulosis into diverticulitis?
1. Treating constipation with chronic laxative use, leading to dependence on
laxatives
2. Chronic constipation causing an obstruction, reducing forward flow of
intestinal contents
3. Herniation of the intestinal mucosa, rupturing the wall of the intestine
4. Undigested food blocking the diverticulum, predisposing the area to bacterial
invasion.

9. Which of the following symptoms indicated diverticulosis?

1. No symptoms exist
2. Change in bowel habits
3. Anorexia with low-grade fever
4. Episodic, dull, or steady midabdominal pain

10. Which of the following tests should be administered to a client


suspected of having diverticulosis?

1. Abdominal ultrasound
2. Barium enema
3. Barium swallow
4. Gastroscopy

11. Medical management of the client with diverticulitis should include


which of the following treatments?

1. Reduced fluid intake


2. Increased fiber in diet
3. Administration of antibiotics
4. Exercises to increase intra-abdominal pressure
12. Crohns disease can be described as a chronic relapsing disease.
Which of the following areas in the GI system may be involved with this
disease?

1. The entire length of the large colon


2. Only the sigmoid area
3. The entire large colon through the layers of mucosa and submucosa
4. The small intestine and colon; affecting the entire thickness of the bowel

13. Which area of the alimentary canal is the most common location for
Crohns disease?

1. Ascending colon
2. Descending colon
3. Sigmoid colon
4. Terminal ileum

14. Which of the following factors is believed to be linked to Crohns


disease?

1. Constipation
2. Diet
3. Hereditary
4. Lack of exercise

15. Which of the following factors is believed to cause ulcerative colitis?

1. Acidic diet
2. Altered immunity
3. Chronic constipation
4. Emotional stress
16. Fistulas are most common with which of the following bowel
disorders?

1. Crohns disease
2. Diverticulitis
3. Diverticulosis
4. Ulcerative colitis

17. Which of the following areas is the most common site of fistulas in
clients with Crohns disease?

1. Anorectal
2. Ileum
3. Rectovaginal
4. Transverse colon

18. Which of the following associated disorders may a client with


ulcerative colitis exhibit?

1. Gallstones
2. Hydronephrosis
3. Nephrolithiasis
4. Toxic megacolon

19. Which of the following associated disorders may the client with
Crohns disease exhibit?

1. Ankylosing spondylitis
2. Colon cancer
3. Malabsorption
4. Lactase deficiency
20. Which of the following symptoms may be exhibited by a client with
Crohns disease?

1. Bloody diarrhea
2. Narrow stools
3. N/V
4. Steatorrhea

21. Which of the following symptoms is associated with ulcerative


colitis?

1. Dumping syndrome
2. Rectal bleeding
3. Soft stools
4. Fistulas

22. If a client had irritable bowel syndrome, which of the following


diagnostic tests would determine if the diagnosis is Crohns disease or
ulcerative colitis?

1. Abdominal computed tomography (CT) scan


2. Abdominal x-ray
3. Barium swallow
4. Colonoscopy with biopsy

23. Which of the following interventions should be included in the


medical management of Crohns disease?

1. Increasing oral intake of fiber


2. Administering laxatives
3. Using long-term steroid therapy
4. Increasing physical activity
24. In a client with Crohns disease, which of the following symptoms
should not be a direct result of antibiotic therapy?

1. Decrease in bleeding
2. Decrease in temperature
3. Decrease in body weight
4. Decrease in the number of stools

25. Surgical management of ulcerative colitis may be performed to treat


which of the following complications?

1. Gastritis
2. Bowel herniation
3. Bowel outpouching
4. Bowel perforation

26. Which of the following medications is most effective for treating


the pain associated with irritable bowel disease?

1. Acetaminophen
2. Opiates
3. Steroids
4. Stool softeners

27. During the first few days of recovery from ostomy surgery for
ulcerative colitis, which of the following aspects should be the first
priority of client care?

1. Body image
2. Ostomy care
3. Sexual concerns
4. Skin care
28. Colon cancer is most closely associated with which of the following
conditions?

1. Appendicitis
2. Hemorrhoids
3. Hiatal hernia
4. Ulcerative colitis

29. Which of the following diets is most commonly associated with


colon cancer?

1. Low-fiber, high fat


2. Low-fat, high-fiber
3. Low-protein, high-carbohydrate
4. Low carbohydrate, high protein

30. Which of the following diagnostic tests should be performed


annually over age 50 to screen for colon cancer?

1. Abdominal CT scan
2. Abdominal x-ray
3. Colonoscopy
4. Fecal occult blood test

31. Radiation therapy is used to treat colon cancer before surgery for
which of the following reasons?

1. Reducing the size of the tumor


2. Eliminating the malignant cells
3. Curing the cancer
4. Helping the bowel heal after surgery
32. Which of the following symptoms is a client with colon cancer most
likely to exhibit?

1. A change in appetite
2. A change in bowel habits
3. An increase in body weight
4. An increase in body temperature

33. A client has just had surgery for colon cancer. Which of the
following disorders might the client develop?

1. Peritonitis
2. Diverticulosis
3. Partial bowel obstruction
4. Complete bowel obstruction

34. A client with gastric cancer may exhibit which of the following
symptoms?

1. Abdominal cramping
2. Constant hunger
3. Feeling of fullness
4. Weight gain

35. Which of the following diagnostic tests may be performed to


determine if a client has gastric cancer?

1. Barium enema
2. Colonoscopy
3. Gastroscopy
4. Serum chemistry levels
36. A client with gastric cancer can expect to have surgery for
resection. Which of the following should be the nursing management
priority for the preoperative client with gastric cancer?

1. Discharge planning
2. Correction of nutritional deficits
3. Prevention of DVT
4. Instruction regarding radiation treatment

37. Care for the postoperative client after gastric resection should focus
on which of the following problems?

1. Body image
2. Nutritional needs
3. Skin care
4. Spiritual needs

38. Which of the following complications of gastric resection should the


nurse teach the client to watch for?

1. Constipation
2. Dumping syndrome
3. Gastric spasm
4. Intestinal spasms

39. A client with rectal cancer may exhibit which of the following
symptoms?

1. Abdominal fullness
2. Gastric fullness
3. Rectal bleeding
4. Right upper quadrant pain
40. A client with which of the following conditions may be likely to
develop rectal cancer?

1. Adenomatous polyps
2. Diverticulitis
3. Hemorrhoids
4. Peptic ulcer disease

41. Which of the following treatments is used for rectal cancer but not
for colon cancer?

1. Chemotherapy
2. Colonoscopy
3. Radiation
4. Surgical resection

42. Which of the following conditions is most likely to directly


cause peritonitis?

1. Cholelithiasis
2. Gastritis
3. Perforated ulcer
4. Incarcerated hernia

43. Which of the following symptoms would a client in the early stages
of peritonitis exhibit?

1. Abdominal distention
2. Abdominal pain and rigidity
3. Hyperactive bowel sounds
4. Right upper quadrant pain
44. Which of the following laboratory results would be expected in a
client with peritonitis?

1. Partial thromboplastin time above 100 seconds


2. Hemoglobin level below 10 mg/dL
3. Potassium level above 5.5 mEq/L
4. White blood cell count above 15,000

45. Which of the following therapies is not included in the medical


management of a client with peritonitis?

1. Broad-spectrum antibiotics
2. Electrolyte replacement
3. I.V. fluids
4. Regular diet

46. Which of the following aspects is the priority focus of nursing


management for a client with peritonitis?

1. Fluid and electrolyte balance


2. Gastric irrigation
3. Pain management
4. Psychosocial issues

47. A client with irritable bowel syndrome is being prepared for


discharge. Which of the following meal plans should the nurse give the
client?

1. Low fiber, low-fat


2. High fiber, low-fat
3. Low fiber, high-fat
4. High-fiber, high-fat
48. A client presents to the emergency room, reporting that he has
been vomiting every 30 to 40 minutes for the past 8 hours. Frequent
vomiting puts him at risk for which of the following?

1. Metabolic acidosis with hyperkalemia


2. Metabolic acidosis with hypokalemia
3. Metabolic alkalosis with hyperkalemia
4. Metabolic alkalosis with hypokalemia

49. Five days after undergoing surgery, a client develops a small-bowel


obstruction. A Miller-Abbott tube is inserted for bowel decompression.
Which nursing diagnosis takes priority?

1. Imbalanced nutrition: Less than body requirements


2. Acute pain
3. Deficient fluid volume
4. Excess fluid volume

50. When teaching an elderly client how to prevent constipation, which


of the following instructions should the nurse include?

1. Drink 6 glasses of fluid each day.


2. Avoid grain products and nuts.
3. Add at least 4 grams of bran to your cereal each morning.
4. Be sure to get regular exercise.

51. In a client with diarrhea, which outcome indicates that fluid


resuscitation is successful?

1. The client passes formed stools at regular intervals


2. The client reports a decrease in stool frequency and liquidity
3. The client exhibits firm skin turgor
4. The client no longer experiences perianal burning.
52. When teaching a community group about measures to prevent colon
cancer, which instruction should the nurse include?

1. Limit fat intake to 20% to 25% of your total daily calories.


2. Include 15 to 20 grams of fiber into your daily diet.
3. Get an annual rectal examination after age 35.
4. Undergo sigmoidoscopy annually after age 50.

53. A 30-year old client experiences weight loss, abdominal distention,


crampy abdominal pain, and intermittent diarrhea after birth of her
2nd child. Diagnostic tests reveal gluten-induced enteropathy. Which
foods must she eliminate from her diet permanently?

1. Milk and dairy products


2. Protein-containing foods
3. Cereal grains (except rice and corn)
4. Carbohydrates

54. After a right hemicolectomy for treatment of colon cancer, a 57-


year old client is reluctant to turn while on bed rest. Which action by
the nurse would be appropriate?

1. Asking a coworker to help turn the client


2. Explaining to the client why turning is important.
3. Allowing the client to turn when hes ready to do so
4. Telling the client that the physicians order states he must turn every 2 hours

55. A client has a percutaneous endoscopic gastrostomy tube inserted


for tube feedings. Before starting a continuous feeding, the nurse
should place the client in which position?

1. Semi-Fowlers
2. Supine
3. Reverse Trendelenburg
4. High Fowlers

56. An enema is prescribed for a client with suspected appendicitis.


Which of the following actions should the nurse take?

1. Prepare 750 ml of irrigating solution warmed to 100*F


2. Question the physician about the order
3. Provide privacy and explain the procedure to the client
4. Assist the client to left lateral Sims position

57. The client being seen in a physicians office has just been scheduled
for a barium swallow the next day. The nurse writes down which of the
following instructions for the client to follow before the test?

1. Fast for 8 hours before the test


2. Eat a regular supper and breakfast
3. Continue to take all oral medications as scheduled.
4. Monitor own bowel movement pattern for constipation

58. The nurse is monitoring a client for the early signs of dumping
syndrome. Which symptom indicates this occurrence?

1. Abdominal cramping and pain


2. Bradycardia and indigestion
3. Sweating and pallor
4. Double vision and chest pain

59. The nurse is preparing a discharge teaching plan for the client who
had an umbilical hernia repair. Which of the following would the nurse
include in the plan?
1. Restricting pain medication
2. Maintaining bedrest
3. Avoiding coughing
4. Irrigating the drain

60. The nurse is caring for a hospitalized client with a diagnosis of


ulcerative colitis. Which finding, if noted on assessment of the client,
would the nurse report to the physician?

1. Bloody diarrhea
2. Hypotension
3. A hemoglobin of 12 mg/dL
4. Rebound tenderness

61. The nurse is reviewing the record of a client with Crohns disease.
Which of the following stool characteristics would the nurse expect to
note documented on the clients record?

1. Chronic constipation
2. Diarrhea
3. Constipation alternating with diarrhea
4. Stool constantly oozing from the rectum

62. The nurse is performing a colostomy irrigation on a client. During


the irrigation, a client begins to complain of abdominal cramps. Which
of the following is the most appropriate nursing action?

1. Notify the physician


2. Increase the height of the irrigation
3. Stop the irrigation temporarily.
4. Medicate with dilaudid and resume the irrigation
63. The nurse is teaching the client how to perform a colostomy
irrigation. To enhance the effectiveness of the irrigation and fecal
returns, what measure should the nurse instruct the client to do?

1. Increase fluid intake


2. Reduce the amount of irrigation solution
3. Perform the irrigation in the evening
4. Place heat on the abdomen

64. The nurse is reviewing the physicians orders written for a client
admitted with acute pancreatitis. Which physician order would the
nurse question if noted on the clients chart?

1. NPO status
2. Insert a nasogastric tube
3. An anticholinergic medication
4. Morphine for pain

65. The nurse is doing an admission assessment on a client with a


history of duodenal ulcer. To determine whether the problem is
currently active, the nurse would assess the client for which of the
following most frequent symptom(s) of duodenal ulcer?

1. Pain that is relieved by food intake


2. Pain that radiated down the right arm
3. N/V
4. Weight loss

66. The nurse instructs the ileostomy client to do which of the following
as a part of essential care of the stoma?

1. Cleanse the peristomal skin meticulously


2. Take in high-fiber foods such as nuts
3. Massage the area below the stoma
4. Limit fluid intake to prevent diarrhea.

67. The client who has undergone creation of a colostomy has a nursing
diagnosis of Disturbed body image. The nurse would evaluate that the
client is making the most significant progress toward identified goals if
the client:

1. Watches the nurse empty the colostomy bag


2. Looks at the ostomy site
3. Reads the ostomy product literature
4. Practices cutting the ostomy appliance

68. The nurse is assessing for stoma prolapse in a client with a


colostomy. The nurse would observe which of the following if stoma
prolapse occurred?

1. Sunken and hidden stoma


2. Dark- and bluish-colored stoma
3. Narrowed and flattened stoma
4. Protruding stoma

69. The client with a new colostomy is concerned about the odor from
the stool in the ostomy drainage bag. The nurse teaches the client to
include which of the following foods in the diet to reduce odor?

1. Yogurt
2. Broccoli
3. Cucumbers
4. Eggs

70. The nurse has given instructions to the client with an ileostomy
about foods to eat to thicken the stool. The nurse determines that the
client needs further instructions if the client stated to eat which of the
following foods to make the stools less watery?

1. Pasta
2. Boiled rice
3. Bran
4. Low-fat cheese

71. The client has just had surgery to create an ileostomy. The nurse
assesses the client in the immediate post-op period for which of the
following most frequent complications of this type of surgery?

1. Intestinal obstruction
2. Fluid and electrolyte imbalance
3. Malabsorption of fat
4. Folate deficiency

72. The nurse is doing pre-op teaching with the client who is about to
undergo creation of a Kock pouch. The nurse interprets that the client
has the best understanding of the nature of the surgery if the client
makes which of the following statements?

1. I will need to drain the pouch regularly with a catheter.


2. I will need to wear a drainage bag for the rest of my life.
3. The drainage from this type of ostomy will be formed.
4. I will be able to pass stool from my rectum eventually.

73. The client with a colostomy has an order for irrigation of the
colostomy. The nurse used which solution for irrigation?

1. Distilled water
2. Tap water
3. Sterile water
4. Lactated Ringers

74. A nurse is monitoring a client admitted to the hospital with a


diagnosis of appendicitis. The client is scheduled for surgery in 2 hours.
The client begins to complain of increased abdominal pain and begins to
vomit. On assessment the nurse notes that the abdomen is distended
and the bowel sounds are diminished. Which of the following is the
most appropriate nursing intervention?

1. Administer dilaudid
2. Notify the physician
3. Call and ask the operating room team to perform the surgery as soon as
possible
4. Reposition the client and apply a heating pad on a warm setting to the
clients abdomen.

75. The client has been admitted with a diagnosis of acute pancreatitis.
The nurse would assess this client for pain that is:

1. Severe and unrelenting, located in the epigastric area and radiating to the
back.
2. Severe and unrelenting, located in the left lower quadrant and radiating to
the groin.
3. Burning and aching, located in the epigastric area and radiating to the
umbilicus.
4. Burning and aching, located in the left lower quadrant and radiating to the
hip.

76. The client with Crohns disease has a nursing diagnosis of acute
pain. The nurse would teach the client to avoid which of the following in
managing this problem?
1. Lying supine with the legs straight
2. Massaging the abdomen
3. Using antispasmodic medication
4. Using relaxation techniques

77. A client with ulcerative colitis has an order to begin salicylate


medication to reduce inflammation. The nurse instructs the client to
take the medication:

1. 30 minutes before meals


2. On an empty stomach
3. After meals
4. On arising

78. During the assessment of a clients mouth, the nurse notes the
absence of saliva. The client is also complaining of pain near the area of
the ear. The client has been NPO for several days because of the
insertion of a NG tube. Based on these findings, the nurse suspects that
the client is developing which of the following mouth conditions?

1. Stomatitis
2. Oral candidiasis
3. Parotitis
4. Gingivitis

79. The nurse evaluates the clients stoma during the initial post-op
period. Which of the following observations should be reported
immediately to the physician?

1. The stoma is slightly edematous


2. The stoma is dark red to purple
3. The stoma oozes a small amount of blood
4. The stoma does not expel stool
80. When planning care for a client with ulcerative colitis who is
experiencing symptoms, which client care activities can the nurse
appropriately delegate to a unlicensed assistant? Select all that apply.

1. Assessing the clients bowel sounds


2. Providing skin care following bowel movements
3. Evaluating the clients response to antidiarrheal medications
4. Maintaining intake and output records
5. Obtaining the clients weight.

81. Which goal of the clients care should take priority during the first
days of hospitalization for an exacerbation of ulcerative colitis?

1. Promoting self-care and independence


2. Managing diarrhea
3. Maintaining adequate nutrition
4. Promoting rest and comfort

82. A clients ulcerative colitis symptoms have been present for longer
than 1 week. The nurse recognizes that the client should be assessed
carefully for signs of which of the following complications?

1. Heart failure
2. DVT
3. Hypokalemia
4. Hypocalcemia

83. A client who has ulcerative colitis has persistent diarrhea. He is thin
and has lost 12 pounds since the exacerbation of his ulcerative colitis.
The nurse should anticipate that the physician will order which of the
following treatment approaches to help the client meet his nutritional
needs?
1. Initiate continuous enteral feedings
2. Encourage a high protein, high-calorie diet
3. Implement total parenteral nutrition
4. Provide six small meals a day.

84. Digoxin preparations and absorbents should not be given


simultaneously. As a nurse, you are aware that if these agents are
given simultaneously, which of the following will occur?

1. Increased absorption of digoxin


2. Decreased absorption of digoxin
3. Increased absorption of the absorbent
4. Decreased absorption of the absorbent

85. When used with hyperacidic disorders of the stomach, antacids are
given to elevate the gastric pH to:

1. 2.0
2. 4.0
3. 6.0
4. >8.0

86. One of your patients is receiving digitalis orally and is also to


receive an antacid at the same time. Your most appropriate action,
based on the pharmacokinetics of antacids, is to:

1. Delay the digitalis for 1 to 2 hours until the antacid is absorbed


2. Give the antacid at least 2 to 4 hours before administering the digitalis
3. Administer both medications as ordered and document in nurses notes
4. Contact the physician regarding the drug interaction and request a change in
the time of dosing of the drugs.
87. The nurse would teach patients that antacids are effective in
treatment of hyperacidity because they:

1. Neutralize gastric acid


2. Decrease stomach motility
3. Decrease gastric pH
4. Decrease duodenal pH

88. The nurse would monitor for which of the following adverse
reactions to aluminum-containing antacids such as aluminum hydroxide
(Amphojel)?

1. Diarrhea
2. Constipation
3. GI upset
4. Fluid retention

89. The nurse would question an order for which type of antacid in
patients with chronic renal failure?

1. Aluminum-containing antacids
2. Calcium-containing antacids
3. Magnesium-containing antacids
4. All of the above.

90. The nurse would monitor a patient using sodium bicarbonate to


treat gastric hyperacidity for signs and symptoms of:

1. Metabolic alkalosis
2. Metabolic acidosis
3. Hyperkalemia
4. Hypercalcemia
91. Which of the following nursing diagnoses is appropriate for a
patient receiving famotidine (Pepcid)?

1. Increased risk for infection due to immunosuppression


2. Potential risk for bleeding related to thrombocytopenia.
3. Alteration in urinary elimination related to retention
4. Alteration in tissue perfusion related to hypertension

92. Histamine2-receptor antagonists:

1. Compete with histamine for binding sites on the parietal cells


2. Irreversibly bind to H+/K+ATPase
3. Cause a decrease in stomach pH
4. Decrease signs and symptoms of allergies related to histamine release

93. Proton pump inhibitors:

1. Gastric ulcer formation


2. GERD
3. Achlorhydria
4. Diverticulosis

94. A patient unable to tolerate oral medications may be prescribed


which of the following proton pump inhibitors to be administered
intravenously?

1. lansoprazole (Prevacid)
2. omeprazole (Prilosec)
3. pantoprazole (Protonix)
4. esomeprazole (Nexium)

95. When administering sucralfate (Carafate) to a patient with a


nasogastric tube, it is important to:
1. Crush the tablet into a fine powder before mixing with water
2. Administer with a bolus tube feeding
3. Allow the tablet to dissolve in water before administering
4. Administer with an antacid for maximum benefit

96. Sucralfate (Carafate) achieves a therapeutic effect by:

1. Neutralizing gastric acid


2. Enhancing gastric absorption
3. Forming a protective barrier around gastric mucosa
4. Inhibiting gastric acid secretion

97. To avoid fecal impaction, psyllium (Metamucil) should be


administered with at least how many ounces of fluid?

1. 4
2. 6
3. 8
4. 10

98. Bismuth subsalicylate (Pepto-Bismol), as an absorbent, has which


of the following mechanisms of action?

1. Decreased GI motility
2. Decreased gastric secretions
3. Increased fluid absorption
4. Binding to diarrhea-causing bacteria for excretion

99. Side effects of loperamide (Imodium) include all of the following


except?

1. Diarrhea
2. epigastric pain
3. Dry mouth
4. Anorexia

100. The mechanism of action of diphenoxylate (Lotomil) is:

1. An increase in intestinal excretion of water


2. An increase in intestinal motility
3. A decrease in peristalsis in the intestinal wall
4. A decrease in the reabsorption of water in the bowel

Answers and Rationale

1. Answer: 1. A fecalith

A fecalith is a fecal calculus, or stone, that occludes the lumen of the appendix
and is the most common cause of appendicitis. Bowel wall swelling, kinking of
the appendix, and external occlusion, not internal occlusion, of the bowel by
adhesions can also be causes of appendicitis.

2. Answer: 4. Steady

The pain begins in the epigastrium or periumbilical region, then shifts to the
right lower quadrant and becomes steady. The pain may be moderate to severe.

3. Answer: 4. Assessing for symptoms of peritonitis

The focus of care is to assess for peritonitis, or inflammation of the peritoneal


cavity. Peritonitis is most commonly caused by appendix rupture and invasion of
bacteria, which could be lethal. The client with appendicitis will have pain that
should be controlled with analgesia. The nurse should discourage oral intake in
preparation for surgery. Discharge teaching is important; however, in the acute
phase, management should focus on minimizing preoperative complications and
recognizing when such may be occurring.
4. Answer: 3. Inflammation of the gastric mucosa

Gastritis is an inflammation of the gastric mucosa that may be acute (often


resulting from exposure to local irritants) or chronic (associated with
autoimmune infections or atrophic disorders of the stomach). Erosion of the
mucosa results in ulceration. Inflammation of a diverticulum is called
diverticulitis; reflux of stomach acid is known as gastroesophageal disease.

5. Answer: 4. Nonsteroidal anti-inflammatory drugs

NSAIDS are a common cause of gastritis because they inhibit prostaglandin


synthesis. Milk, once thought to help gastritis, has little effect on the stomach
mucosa. Bicarbonate of soda, or baking soda, may be used to neutralize
stomach acid, but it should be used cautiously because it may lead to metabolic
acidosis. ASA with enteric coating shouldnt contribute significantly to gastritis
because the coating limits the aspirins effect on the gastric mucosa.

6. Answer: 2. A noninflamed outpouching of the intestine

Diverticulosis involves a noninflamed outpouching of the intestine. Diverticulitis


involves an inflamed outpouching. The partial impairment of forward flow of the
intestine is an obstruction; abnormal protrusion of an organ is a hernia.

7. Answer: 1. Low-fiber diet

Low-fiber diets have been implicated in the development of diverticula because


these diets decrease the bulk in the stool and predispose the person to the
development of constipation. A high-fiber diet is recommended to help prevent
diverticulosis. A high-protein or low-carbohydrate diet has no effect on the
development of diverticulosis.

8. Answer: 4. Undigested food blocking the diverticulum, predisposing


the area to bacterial invasion.
Undigested food can block the diverticulum, decreasing blood supply to the area
and predisposing the area to invasion of bacteria. Chronic laxative use is a
common problem in elderly clients, but it doesnt cause diverticulitis. Chronic
constipation can cause an obstructionnot diverticulitis. Herniation of the
intestinal mucosa causes an intestinal perforation.

9. Answer: 1. No symptoms exist

Diverticulosis is an asymptomatic condition. The other choices are signs and


symptoms of diverticulitis.

10. Answer: 2. Barium enema

A barium enema will cause diverticula to fill with barium and be easily seen on
x-ray. An abdominal US can tell more about structures, such as the gallbladder,
liver, and spleen, than the intestine. A barium swallow and gastroscopy view
upper GI structures.

11. Answer: 3. Administration of antibiotics

Antibiotics are used to reduce the inflammation. The client isnt typically isnt
allowed anything orally until the acute episode subsides. Parenteral fluids are
given until the client feels better; then its recommended that the client drink
eight 8-ounce glasses of water per day and gradually increase fiber in the diet
to improve intestinal motility. During the acute phase, activities that increase
intra-abdominal pressure should be avoided to decrease pain and the chance of
intestinal obstruction.

12. Answer: 4. The small intestine and colon; affecting the entire
thickness of the bowel
Crohns disease can involve any segment of the small intestine, the colon, or
both, affecting the entire thickness of the bowel. Answers 1 and 3 describe
ulcerative colitis, answer 2 is too specific and therefore, not likely.

13. Answer: 4. Terminal ileum

Studies have shown that the terminal ileum is the most common site for
recurrence in clients with Crohns disease. The other areas may be involved but
arent as common.

14. Answer: 3. Hereditary

Although the definite cause of Crohns disease is unknown, its thought to be


associated with infectious, immune, or psychological factors. Because it has a
higher incidence in siblings, it may have a genetic cause.

15. Answer: 2. Altered immunity

Several theories exist regarding the cause of ulcerative colitis. One suggests
altered immunity as the cause based on the extraintestinal characteristics of the
disease, such as peripheral arthritis and cholangitis. Diet and constipation have
no effect on the development of ulcerative colitis. Emotional stress can
exacerbate the attacks but isnt believed to be the primary cause.

16. Answer: 1. Crohns disease

The lesions of Crohns disease are transmural; that is, they involve all thickness
of the bowel. These lesions may perforate the bowel wall, forming fistulas with
adjacent structures. Fistulas dont develop in diverticulitis or diverticulosis. The
ulcers that occur in the submucosal and mucosal layers of the intestine in
ulcerative colitis usually dont progress to fistula formation as in Crohns
disease.
17. Answer: 1. Anorectal

Fistulas occur in all these areas, but the anorectal area is most common
because of the relative thinness of the intestinal wall in this area.

18. Answer: 4. Toxic megacolon

Toxic megacolon is extreme dilation of a segment of the diseased colon caused


by paralysis of the colon, resulting in complete obstruction. This disorder is
associated with both Crohns disease and ulcerative colitis. The other disorders
are more commonly associated with Crohns disease.

19. Answer: 3. Malabsorption

Because of the transmural nature of Crohns disease lesions, malabsorption may


occur with Crohns disease. Ankylosing spondylitis and colon cancer are more
commonly associated with ulcerative colitis. Lactase deficiency is caused by a
congenital defect in which an enzyme isnt present.

20. Answer: 4. Steatorrhea

Steatorrhea from malabsorption can occur with Crohns disease. N/V, and
bloody diarrhea are symptoms of ulcerative colitis. Narrow stools are associated
with diverticular disease.

21. Answer: 2. Rectal bleeding

In ulcerative colitis, rectal bleeding is the predominant symptom. Soft stools are
more commonly associated with Crohns disease, in which malabsorption is
more of a problem. Dumping syndrome occurs after gastric surgeries. Fistulas
are associated with Crohns disease.

22. Answer: 4. Colonoscopy with biopsy


A colonoscopy with biopsy can be performed to determine the state of the
colons mucosal layers, presence of ulcerations, and level of cytologic
development. An abdominal x-ray or CT scan wouldnt provide the cytologic
information necessary to diagnose which disease it is. A barium swallow doesnt
involve the intestine.

23. Answer: 3. Using long-term steroid therapy

Management of Crohns disease may include long-term steroid therapy to


reduce the inflammation associated with the deeper layers of the bowel wall.
Other management focuses on bowel rest (not increasing oral intake) and
reducing diarrhea with medications (not giving laxatives). The pain associated
with Crohns disease may require bed rest, not an increase in physical activity.

24. Answer: 3. Decrease in body weight

A decrease in body weight may occur during therapy due to inadequate dietary
intake, but isnt related to antibiotic therapy. Effective antibiotic therapy will be
noted by a decrease in temperature, number of stools, and bleeding.

25. Answer: 4. Bowel perforation

Perforation, obstruction, hemorrhage, and toxic megacolon are common


complications of ulcerative colitis that may require surgery. Herniation and
gastritis arent associated with irritable bowel diseases, and outpouching of the
bowel is diverticulosis.

26. Answer: 3. Steroids

The pain with irritable bowel disease is caused by inflammation, which steroids
can reduce. Stool softeners arent necessary. Acetaminophen has little effect on
the pain, and opiate narcotics wont treat its underlying cause.
27. Answer: 2. Ostomy care

Although all of these are concerns the nurse should address, being able to
safely manage the ostomy is crucial for the client before discharge.

28. Answer: 4. Ulcerative colitis

Chronic ulcerative colitis, granulomas, and familial polyposis seem to increase a


persons chance of developing colon cancer. The other conditions listed have no
known effect on colon cancer risk.

29. Answer: 1. Low-fiber, high fat

A low-fiber, high-fat diet reduced motility and increases the chance of


constipation. The metabolic end products of this type of diet are carcinogenic. A
low-fat, high-fiber diet is recommended to prevent colon cancer.

30. Answer: 4. Fecal occult blood test

Surface blood vessels of polyps and cancers are fragile and often bleed with the
passage of stools. Abdominal x-ray and CT scan can help establish tumor size
and metastasis. A colonoscopy can help locate a tumor as well as polyps, which
can be removed before they become malignant.

31. Answer: 1. Reducing the size of the tumor

Radiation therapy is used to treat colon cancer before surgery to reduce the size
of the tumor, making it easier to be resected. Radiation therapy isnt curative,
cant eliminate the malignant cells (though it helps define tumor margins), can
could slow postoperative healing.

32. Answer: 2. A change in bowel habits


The most common complaint of the client with colon cancer is a change in bowel
habits. The client may have anorexia, secondary abdominal distention, or
weight loss. Fever isnt associated with colon cancer.

33. Answer: 1. Peritonitis

Bowel spillage could occur during surgery, resulting in peritonitis. Complete or


partial bowel obstruction may occur before bowel resection. Diverticulosis
doesnt result from surgery or colon cancer.

34. Answer: 3. Feeling of fullness

The client with gastric cancer may report a feeling of fullness in the stomach,
but not enough to cause him to seek medical attention. Abdominal cramping
isnt associated with gastric cancer. Anorexia and weight loss (not increased
hunger or weight gain) are common symptoms of gastric cancer.

35. Answer: 3. Gastroscopy

A gastroscopy will allow direct visualization of the tumor. A colonoscopy or a


barium enema would help diagnose colon cancer. Serum chemistry levels dont
contribute data useful to the assessment of gastric cancer.

36. Answer: 2. Correction of nutritional deficits

Clients with gastric cancer commonly have nutritional deficits and may be
cachectic. Discharge planning before surgery is important, but correcting the
nutrition deficit is a higher priority. At present, radiation therapy hasnt been
proven effective for gastric cancer, and teaching about it preoperatively wouldnt
be appropriate. Prevention of DVT also isnt a high priority to surgery, though it
assumes greater importance after surgery.

37. Answer: 2. Nutritional needs


After gastric resection, a client may require total parenteral nutrition or
jejunostomy tube feedings to maintain adequate nutritional status.

38. Answer: 2. Dumping syndrome

Dumping syndrome is a problem that occurs postprandially after gastric


resection because ingested food rapidly enters the jejunum without proper
mixing and without the normal duodenal digestive processing. Diarrhea, not
constipation, may also be a symptom. Gastric or intestinal spasms dont occur,
but antispasmodics may be given to slow gastric emptying.

39. Answer: 3. Rectal bleeding

Rectal bleeding is a common symptom of rectal cancer. Rectal cancer may be


missed because other conditions such as hemorrhoids can cause rectal bleeding.
Abdominal fullness may occur with colon cancer, gastric fullness may occur with
gastric cancer, and right upper quadrant pain may occur with liver cancer.

40. Answer: 1. Adenomatous polyps

A client with adenomatous polyps has a higher risk for developing rectal cancer
than others do. Clients with diverticulitis are more likely to develop colon
cancer. Hemorrhoids dont increase the chance of any type of cancer. Clients
with peptic ulcer disease have a higher incidence of gastric cancer.

41. Answer: 3. Radiation

A client with rectal cancer can expect to have radiation therapy in addition
to chemotherapy and surgical resection of the tumor. A colonoscopy is
performed to diagnose the disease. Radiation therapy isnt usually indicated in
colon cancer.

42. Answer: 3. Perforated ulcer


The most common cause of peritonitis is a perforated ulcer, which can pour
contaminates into the peritoneal cavity, causing inflammation
and infection within the cavity. The other conditions dont by themselves
cause peritonitis. However, if cholelithiasis leads to rupture of the gallbladder,
gastritis leads to erosion of the stomach wall, or an incarcerated hernia leads to
rupture of the intestines, peritonitis may develop.

43. Answer: 2. Abdominal pain and rigidity

Abdominal pain causing rigidity of the abdominal muscles is characteristic of


peritonitis. Abdominal distention may occur as a late sign but not early on.
Bowel sounds may be normal or decreased but not increased. Right upper
quadrant pain is characteristic of cholecystitis or hepatitis.

44. Answer: 4. White blood cell count above 15,000

Because of infection, the clients WBC count will be elevated. A hemoglobin level
below 10 mg/dl may occur from hemorrhage. A PT time longer than 100
seconds may suggest disseminated intravascular coagulation, a serious
complication of septic shock. A potassium level above 5.5 mEq/L may indicate
renal failure.

45. Answer: 4. Regular diet

The client with peritonitis usually isnt allowed anything orally until the source of
peritonitis is confirmed and treated. The client also requires broad-spectrum
antibiotics to combat the infection. I.V. fluids are given to maintain hydration
and hemodynamic stability and to replace electrolytes.

46. Answer: 1. Fluid and electrolyte balance

Peritonitis can advance to shock and circulatory failure, so fluid and electrolyte
balance is the priority focus of nursing management. Gastric irrigation may be
needed periodically to ensure patency of the nasogastric tube. Although pain
management is important for comfort and psychosocial care will address
concerns such as anxiety, focusing on fluid and electrolyte imbalance will
maintain hemodynamic stability.

47. Answer: 2. High-fiber, low-fat

The client with irritable bowel syndrome needs to be on a diet that contains at
least 25 grams of fiber per day. Fatty foods are to be avoided because they may
precipitate symptoms.

48. Answer: 4. Metabolic alkalosis with hypokalemia

Gastric acid contains large amounts of potassium, chloride, and hydrogen ions.
Excessive loss of these substances, such as from vomiting, can lead to
metabolic alkalosis and hypokalemia.

49. Answer: 3. Deficient fluid volume

Fluid shifts to the site of the bowel obstruction, causing a fluid deficit in the
intravascular spaces. If the obstruction isnt resolved immediately, the client
may experience an imbalanced nutritional status (less than body requirements);
however, deficient fluid volume takes priority. The client may also experience
pain, but that nursing diagnosis is also of lower priority than deficient fluid
volume.

50. Answer: 4. Be sure to get regular exercise.

Exercise helps prevent constipation. Fluids and dietary fiber promote normal
bowel function. The client should drink eight to ten glasses of fluid each day.
Although adding bran to cereal helps prevent constipation by increasing dietary
fiber, the client should start with a small amount and gradually increase the
amount as tolerated to a maximum of 2 grams a day.
51. Answer: 3. The client exhibits firm skin turgor

A client with diarrhea has a nursing diagnosis of Deficient fluid volume related
to excessive fluid loss in the stool. Expected outcomes include firm skin turgor,
moist mucous membranes, and urine output of at least 30 ml/hr. The client also
has a nursing diagnosis of diarrhea, with expected outcomes of passage of
formed stools at regular intervals and a decrease in stool frequency and
liquidity. The client is at risk for impaired skin integrity related to irritation from
diarrhea; expected outcomes for this diagnosis include absence of erythema in
perianal skin and mucous membranes and absence of perianal tenderness or
burning.

52. Answer: 1. Limit fat intake to 20% to 25% of your total daily
calories.

To help prevent colon cancer, fats should account for no more than 20% to 25%
of total daily calories and the diet should include 25 to 30 grams of fiber per
day. A digital rectal examination isnt recommended as a stand-alone test for
colorectal cancer. For colorectal cancer screening, the American Cancer society
advises clients over age 50 to have a flexible sigmoidoscopy every 5 years,
yearly fecal occult blood tests, yearly fecal occult blood tests PLUS a flexible
sigmoidoscopy every 5 years, a double-contrast barium enema every 5 years,
or a colonoscopy every 10 years.

53. Answer: 3. Cereal grains (except rice and corn)

To manage gluten-induced enteropathy, the client must eliminate gluten, which


means avoiding all cereal grains except for rice and corn. In initial disease
management, clients eat a high calorie, high-protein diet with mineral and
vitamin supplements to help normalize nutritional status.

54. Answer: 2. Explaining to the client why turning is important.


The appropriate action is to explain the importance of turning to avoid
postoperative complications. Asking a coworker to help turn the client would
infringe on his rights. Allowing him to turn when hes ready would increase his
risk for postoperative complications. Telling him he must turn because of the
physicians orders would put him on the defensive and exclude him from
participating in care decision.

55. Answer: 1. Semi-Fowlers

To prevent aspiration of stomach contents, the nurse should place the client in
semi-Fowlers position. High Fowlers position isnt necessary and may not be
tolerated as well as semi-Fowlers.

56. Answer: 2. Question the physician about the order

Enemas are contraindicated in an acute abdominal condition of unknown origin


as well as after recent colon or rectal surgery or myocardial infarction. The
other answers are correct only when enema administration is appropriate.

57. Answer: 1. Fast for 8 hours before the test

A barium swallow is an x-ray study that uses a substance called barium for
contrast to highlight abnormalities in the GI tract. The client should fast for 8 to
12 hours before the test, depending on the physician instructions. Most oral
medications also are withheld before the test. After the procedure the nurse
must monitor for constipation, which can occur as a result of the presence of
barium in the GI tract.

58. Answer: 3. Sweating and pallor

Early manifestations of dumping syndrome occur 5 to 30 minutes after eating.


Symptoms include vertigo, tachycardia, syncope, sweating, pallor, palpitations,
and the desire to lie down.
59. Answer: 3. Avoiding coughing

Bedrest is not required following this surgical procedure. The client should take
analgesics as needed and as prescribed to control pain. A drain is not used in
this surgical procedure, although the client may be instructed in simple dressing
changes. Coughing is avoided to prevent disruption of the tissue integrity, which
can occur because of the location of this surgical procedure.

60. Answer: 4. Rebound tenderness

Rebound tenderness may indicate peritonitis. Blood diarrhea is expected to


occur in ulcerative colitis. Because of the blood loss, the client may be
hypotensive and the hemoglobin level may be lower than normal. Signs of
peritonitis must be reported to the physician.

61. Answer: 2. Diarrhea

Crohns disease is characterized by nonbloody diarrhea of usually not more than


four to five stools daily. Over time, the diarrhea episodes increase in frequency,
duration and severity. The other options are not associated with diarrhea.

62. Answer: 3. Stop the irrigation temporarily.

If cramping occurs during a colostomy irrigation, the irrigation flow is stopped


temporarily and the client is allowed to rest. Cramping may occur from an
infusion that is too rapid or is causing too much pressure. Increasing the height
of the irrigation will cause further discomfort. The physician does not need to be
notified. Medicating the client for pain is not the most appropriate action.

63. Answer: 1. Increase fluid intake

To enhance effectiveness of the irrigation and fecal returns, the client is


instructed to increase fluid intake and prevent constipation.
64. Answer: 4. Morphine for pain

Meperidine (Demerol) rather than morphine is the medication of choice because


morphine can cause spasm in the sphincter of Oddi.

65. Answer: 1. Pain that is relieved by food intake

The most frequent symptom of duodenal ulcer is pain that is relieved by food
intake. These clients generally describe the pain as burning, heavy, sharp, or
hungry pain that often localizes in the midepigastric area. The client with
duodenal ulcer usually does not experience weight loss or N/V. These symptoms
are usually more typical in the client with a gastric ulcer.

66. Answer: 1. Cleanse the peristomal skin meticulously

The peristomal skin must receive meticulous cleansing because the ileostomy
drainage has more enzymes and is more caustic to the skin than colostomy
drainage. Foods such as nuts and those with seeds will pass through the
ileostomy. The client should be taught that these foods will remain undigested.
The area below the ileostomy may be massaged if needed if the ileostomy
becomes blocked by high fiber foods. Fluid intake should be maintained to at
least six to eight glasses of water per day to prevent dehydration.

67. Answer: 4. Practices cutting the ostomy appliance

The client is expected to have a body image disturbance after colostomy. The
client progresses through normal grieving stages to adjust to this change. The
client demonstrates the greatest deal of acceptance when the client participates
in the actual colostomy care. Each of the incorrect options represents an
interest in colostomy care but is a passive activity. The correct option shows the
client is participating in self-care.

68. Answer: 4. Protruding stoma


A prolapsed stoma is one which the bowel protruded through the stoma. A
stoma retraction is characterized by sinking of the stoma. Ischemia of the
stoma would be associated with dusky or bluish color. A stoma with a narrowed
opening at the level of the skin or fascia is said to be stenosed.

69. Answer: 1. Yogurt

The client should be taught to include deodorizing foods in the diet, such a beet
greens, parsley, buttermilk, and yogurt. Spinach also reduces odor but is a gas
forming food as well. Broccoli, cucumbers, and eggs are gas forming foods.

70. Answer: 3. Bran

Foods that help thicken the stool of the client with an ileostomy include pasta,
boiled rice, and low-fat cheese. Bran is high in dietary fiber and thus will
increase output of watery stool by increasing propulsion through the bowel.
Ileostomy output is liquid. Addition or elimination of various foods can help
thicken or loosen this liquid drainage.

71. Answer: 2. Fluid and electrolyte imbalance

A major complication that occurs most frequent following an ileostomy is fluid


and electrolyte imbalance. The client requires constant monitoring of intake and
output to prevent this from happening. Losses require replacement by
intravenous infusion until the client can tolerate a diet orally. Intestinal
obstruction is a less frequent complication. Fat malabsorption and folate
deficiency are complications that could occur later in the postoperative period.

72. Answer: 1. I will need to drain the pouch regularly with a


catheter.

A Kock pouch is a continent ileostomy. As the ileostomy begins to function, the


client drains it every 3 to 4 hours and then decreases the draining to about 3
times a day or as needed when full. The client does not need to wear a drainage
bag but should wear an absorbent dressing to absorb mucus drainage from the
stoma. Ileostomy drainage is liquid. The client would be able to pass stool only
from the rectum if an ileal-anal pouch or anastomosis were created. This type of
operation is a two-stage procedure.

73. Answer: 2. Tap water

Warm tap water or saline solution is used to irrigate a colostomy. If the tap
water is not suitable for drinking, then bottled water should be used.

74. Answer: 2. Notify the physician

Based on the signs and symptoms presented in the question, the nurse should
suspect peritonitis and should notify the physician. Administering pain
medication is not an appropriate intervention. Heat should never be applied to
the abdomen of a client with suspected appendicitis. Scheduling surgical time is
not within the scope of nursing practice, although the physician probably would
perform the surgery earlier than the prescheduled time.

75. Answer: 1. Severe and unrelenting, located in the epigastric area


and radiating to the back.

The pain associated with acute pancreatitis is often severe and unrelenting, is
located in the epigastric region, and radiates to the back.

76. Answer: 1. Lying supine with the legs straight

The pain associated with Crohns disease is alleviated by the use of analgesics
and antispasmodics and also is reduced by having the client practice relaxation
techniques, applying local cold or heat to the abdomen, massaging the
abdomen, and lying with the legs flexed. Lying with the legs extended is not
useful because it increases the muscle tension in the abdomen, which could
aggravate the inflamed intestinal tissues as the abdominal muscles are
stretched.

77. Answer: 3. After meals

Salicylate compounds act by inhibiting prostaglandin synthesis and reducing


inflammation. The nurse teaches the client to take the medication with a full
glass of water and to increase fluid intake throughout the day. This medication
needs to be taken after meals to reduce GI irritation.

78. Answer: 3. Parotitis

The lack of saliva, pain near the area of the ear, and the prolonged NPO status
of the client should lead the nurse to suspect the development of parotitis, or
inflammation of the parotid gland. Parotitis usually develops in cases
of dehydration combined with poor oral hygiene or when clients have been NPO
for an extended period. Preventative measures include the use of sugarless hard
candy or gum to stimulate saliva production, adequate hydration, and frequent
mouth care. Stomatitis (inflammation of the mouth) produces excessive
salivation and a sore mouth.

79. Answer: 2. The stoma is dark red to purple

A dark red to purple stoma indicates inadequate blood supply. Mild edema and
slight oozing of blood are normal in the early post-op period. The colostomy
would typically not begin functioning until 2-4 days after surgery.

80. Answer: 2, 4, and 5.

The nurse can delegate the following basic care activities to the unlicensed
assistant: providing skin care following bowel movements, maintaining intake
and output records, and obtaining the clients weight. Assessing the clients
bowel sounds and evaluating the clients response to medication are registered
nurse activities that cannot be delegated.

81. Answer: 2. Managing diarrhea

Diarrhea is the primary symptom in an exacerbation of ulcerative colitis, and


decreasing the frequency of stools is the first goal of treatment. The other goals
are ongoing and will be best achieved by halting the exacerbation. The client
may receive antidiarrheal medications, antispasmodic agents, bulk hydrophilic
agents, or anti-inflammatory drugs.

82. Answer: 3. Hypokalemia

Excessive diarrhea causes significant depletion of the bodys stores of sodium


and potassium as well as fluid. The client should be closely monitored for
hypokalemia and hyponatremia. Ulcerative colitis does not place the client at
risk for heart failure, DVT, or hypocalcemia.

83. Answer: 3. Implement total parenteral nutrition

Food will be withheld from the client with severe symptoms of ulcerative colitis
to rest the bowel. To maintain the clients nutritional status, the client will be
started on TPN. Enteral feedings or dividing the diet into 6 small meals does not
allow the bowel to rest. A high-calorie, high-protein diet will worsen the clients
symptoms.

84. Answer: 2. Decreased absorption of digoxin

85. Answer: 1. 2.0

86. Answer: 4. Contact the physician regarding the drug interaction and
request a change in the time of dosing of the drugs.
87. Answer: 1. Neutralize gastric acid

Antacids work by neutralizing gastric acid, which would cause an increase in pH.
They do not affect gastric motility.

88. Answer: 2. Constipation

Aluminum- and calcium-containing antacids cause constipation, magnesium-


containing antacids cause diarrhea and sodium-containing antacids cause
sodium and fluid retention.

89. Answer: 3. Magnesium-containing antacids

Magnesium-containing antacids can cause hypermagnesemia in patients


with chronic renal failure. Aluminum-containing antacids may be used as a
phosphate binder in patients with chronic renal failure. Calcium-containing
antacids are also appropriate because these patients may be hypocalcemic.

90. Answer: 1. Metabolic alkalosis

Solutions containing sodium bicarbonate (a base) can cause metabolic alkalosis.


Serum K and serum calcium would decrease with alkalosis, not increase.

91. Answer: 2. Potential risk for bleeding related to thrombocytopenia.

A serious side effect of famotidine is thrombocytopenia, which is manifested by


a decrease in platelet count and an increased risk of bleeding.

92. Answer: 1. Compete with histamine for binding sites on the parietal
cells

Histamine receptor blocking agents decrease gastric acid by competing with


histamine for binding sites on the parietal cells.
93. Answer: 3. Achlorhydria

Because the proton pump inhibitors stop the final step of acid secretion, they
can block up to 90% of acid secretion, leading to achlorhydria (without acid).

94. Answer: 3. pantoprazole (Protonix)

Pantoprazole is the only proton pump inhibitor that is available for intravenous
administration. The other medications in this category may only be
administered orally.

95. Answer: 3. Allow the tablet to dissolve in water before


administering

It is important to give sucralfate on an empty stomach so that it may dissolve


and form a protective barrier over the gastric mucosa. The tablet form will not
dissolve in water when crushed; it must be left whole and allowed to dissolve.
Crushing the medication so that it will not dissolve could lead to clogging of the
nasogastric tube and decreased effectiveness of the drug.

96. Answer: 3. Forming a protective barrier around gastric mucosa

Sucralfate has a local effect only on the gastric mucosa. It forms a paste-like
substance in the stomach, which adheres to the gastric lining, protecting
against adverse effects related to gastric acid. It also stimulates healing of any
ulcerated areas of the gastric mucosa.

97. Answer: 3. 8

Bulk-forming laxatives must be given with at least 8 ounces of liquid plus


additional liquid each day to prevent intestinal obstruction.

98. Answer: 4. Binding to diarrhea-causing bacteria for excretion


Absorbent antidiarrheal medications bind to diarrhea-causing bacteria to form a
nonabsorbable complex, which is then excreted in the stool.

99. Answer: 1. Diarrhea

Side effects associated with loperamide include CNS fatigue and dizziness,
epigastric pain, abdominal cramps, nausea, dry mouth, vomiting, and anorexia.
Diarrhea is an indication, not a side effect.

100. Answer: 3. A decrease in peristalsis in the intestinal wall

Diphenoxylate acts on the smooth muscle of the intestinal tract to inhibit GI


motility and excessive propulsion of the GI tract (peristalsis).

1. Which of the following conditions can cause a hiatal hernia?

1. Increased intrathoracic pressure


2. Weakness of the esophageal muscle
3. Increased esophageal muscle pressure
4. Weakness of the diaphragmatic muscle

2. Risk factors for the development of hiatal hernias are those that lead
to increased abdominal pressure. Which of the following complications
can cause increased abdominal pressure?

1. Obesity
2. Volvulus
3. Constipation
4. Intestinal obstruction

3. Which of the following symptoms is common with a hiatal hernia?


1. Left arm pain
2. Lower back pain
3. Esophageal reflux
4. Abdominal cramping

4. Which of the following tests can be performed to diagnose a hiatal


hernia?

1. Colonoscopy
2. Lower GI series
3. Barium swallow
4. Abdominal x-rays

5. Which of the following measures should the nurse focus on for the
client with esophageal varices?

1. Recognizing hemorrhage
2. Controlling blood pressure
3. Encouraging nutritional intake
4. Teaching the client about varices

6. Which of the following tests can be used to diagnose ulcers?

1. Abdominal x-ray
2. Barium swallow
3. Computed tomography (CT) scan
4. Esophagogastroduodenoscopy (EGD)

7. Which of the following best describes the method of action of


medications, such as ranitidine (Zantac), which are used in the
treatment of peptic ulcer disease?
1. Neutralize acid
2. Reduce acid secretions
3. Stimulate gastrin release
4. Protect the mucosal barrier

8. The hospitalized client with GERD is complaining of chest discomfort


that feels like heartburn following a meal. After administering an
ordered antacid, the nurse encourages the client to lie in which of the
following positions?

1. Supine with the head of the bed flat


2. On the stomach with the head flat
3. On the left side with the head of the bed elevated 30 degrees
4. On the right side with the head of the bed elevated 30 degrees.

9. The nurse is caring for a client following a Billroth II procedure. On


review of the post-operative orders, which of the following, if
prescribed, would the nurse question and verify?

1. Irrigating the nasogastric tube


2. Coughing a deep breathing exercises
3. Leg exercises
4. Early ambulation

10. The nurse is providing discharge instructions to a client following


gastrectomy. Which measure will the nurse instruct the client to follow
to assist in preventing dumping syndrome?

1. Eat high-carbohydrate foods


2. Limit the fluids taken with meals
3. Ambulate following a meal
4. Sit in a high-Fowlers position during meals
11. The nurse instructs the nursing assistant on how to provide oral
hygiene for a client who cannot perform this task for himself. Which of
the following techniques should the nurse tell the assistant to
incorporate into the clients daily care?

1. Assess the oral cavity each time mouth care is given and record observations
2. Use a soft toothbrush to brush the clients teeth after each meal
3. Swab the clients tongue, gums, and lips with a soft foam applicator every 2
hours.
4. Rinse the clients mouth with mouthwash several times a day.

12. A client with suspected gastric cancer undergoes an endoscopy of


the stomach. Which of the following assessments made after the
procedure would indicate the development of a potential complication?

1. The client complains of a sore throat


2. The client displays signs of sedation
3. The client experiences a sudden increase in temperature
4. The client demonstrates a lack of appetite

13. A client has been diagnosed with adenocarcinoma of the stomach


and is scheduled to undergo a subtotal gastrectomy (Billroth II
procedure). During pre-operative teaching, the nurse is reinforcing
information about the procedure. Which of the following explanations is
most accurate?

1. The procedure will result in enlargement of the pyloric sphincter


2. The procedure will result in anastomosis of the gastric stump to the jejunum
3. The procedure will result in removal of the duodenum
4. The procedure will result in repositioning of the vagus nerve
14. After a subtotal gastrectomy, the nurse should anticipate
that nasogastric tube drainage will be what color for about 12 to 24
hours after surgery?

1. Dark brown
2. Bile green
3. Bright red
4. Cloudy white

15. After a subtotal gastrectomy, care of the clients nasogastric


tube and drainage system should include which of the following nursing
interventions?

1. Irrigate the tube with 30 ml of sterile water every hour, if needed.


2. Reposition the tube if it is not draining well
3. Monitor the client for N/V, and abdominal distention
4. Turn the machine to high suction of the drainage is sluggish on low suction.

16. Which of the following would be an expected nutritional outcome


for a client who has undergone a subtotal gastrectomy for cancer?

1. Regain weight loss within 1 month after surgery


2. Resume normal dietary intake of three meals per day
3. Control nausea and vomiting through regular use of antiemetics
4. Achieve optimal nutritional status through oral or parenteral feedings

17. The client with GERD complains of a chronic cough. The nurse
understands that in a client with GERD this symptom may be indicative
of which of the following conditions?

1. Development of laryngeal cancer


2. Irritation of the esophagus
3. Esophageal scar tissue formation
4. Aspiration of gastric contents

18. Which of the following dietary measures would be useful in


preventing esophageal reflux?

1. Eating small, frequent meals


2. Increasing fluid intake
3. Avoiding air swallowing with meals
4. Adding a bedtime snack to the dietary plan

19. A client is admitted to the hospital after vomiting bright red blood
and is diagnosed with a bleeding duodenal ulcer. The client develops a
sudden, sharp pain in the mid epigastric area along with a rigid, board-
like abdomen. These clinical manifestations most likely indicate which
of the following?

1. An intestinal obstruction has developed


2. Additional ulcers have developed
3. The esophagus has become inflamed
4. The ulcer has perforated

20. When obtaining a nursing history on a client with a suspected


gastric ulcer, which signs and symptoms would the nurse expect to
see? Select all that apply.

1. Epigastric pain at night


2. Relief of epigastric pain after eating
3. Vomiting
4. Weight loss
21. The nurse is caring for a client who has had a gastroscopy. Which of
the following symptoms may indicate that the client is developing a
complication related to the procedure? Select all that apply.

1. The client complains of a sore throat


2. The client has a temperature of 100*F
3. The client appears drowsy following the procedure
4. The client complains of epigastric pain
5. The client experiences hematemesis

22. A client with peptic ulcer disease tells the nurse that he has black
stools, which he has not reported to his physician. Based on this
information, which nursing diagnosis would be appropriate for this
client?

1. Ineffective coping related to fear of diagnosis of chronic illness


2. Deficient knowledge related to unfamiliarity with significant signs and
symptoms
3. Constipation related to decreased gastric motility
4. Imbalanced nutrition: Less than body requirements due to gastric bleeding

23. A client with a peptic ulcer reports epigastric pain that frequently
awakens her at night, a feeling of fullness in the abdomen, and a
feeling of anxiety about her health. Based on this information, which
nursing diagnosis would be most appropriate?

1. Imbalanced Nutrition: Less than Body Requirements related to anorexia.


2. Disturbed Sleep Pattern related to epigastric pain
3. Ineffective Coping related to exacerbation of duodenal ulcer
4. Activity Intolerance related to abdominal pain

24. While caring for a client with peptic ulcer disease, the client reports
that he has been nauseated most of the day and is now feeling
lightheaded and dizzy. Based upon these findings, which nursing
actions would be most appropriate for the nurse to take? Select all that
apply.

1. Administering an antacid hourly until nausea subsides.


2. Monitoring the clients vital signs
3. Notifying the physician of the clients symptoms
4. Initiating oxygen therapy
5. Reassessing the client on an hour

25. A client is to take one daily dose of ranitidine (Zantac) at home to


treat her peptic ulcer. The nurse knows that the client understands
proper drug administration of ranitidine when she says that she will
take the drug at which of the following times?

1. Before meals
2. With meals
3. At bedtime
4. When pain occurs

26. A client has been taking aluminum hydroxide 30 mL six times per
day at home to treat his peptic ulcer. He tells the nurse that he has
been unable to have a bowel movement for 3 days. Based on this
information, the nurse would determine that which of the following is
the most likely cause of the clients constipation?

1. The client has not been including enough fiber in his diet
2. The client needs to increase his daily exercise
3. The client is experiencing a side effect of the aluminum hydroxide.
4. The client has developed a gastrointestinal obstruction.
27. A client is taking an antacid for treatment of a peptic ulcer. Which of
the following statements best indicates that the client understands how
to correctly take the antacid?

1. I should take my antacid before I take my other medications.


2. I need to decrease my intake of fluids so that I dont dilute the effects of my
antacid.
3. My antacid will be most effective if I take it whenever I experience stomach
pains.
4. It is best for me to take my antacid 1 to 3 hours after meals.

28. The nurse is caring for a client with chronic gastritis. The nurse
monitors the client, knowing that this client is at risk for which of the
following vitamin deficiencies?

1. Vitamin A
2. Vitamin B12
3. Vitamin C
4. Vitamin E

29. The nurse is reviewing the medication record of a client with acute
gastritis. Which medication, if noted on the clients record, would the
nurse question?

1. Digoxin (Lanoxin)
2. Indomethacin (Indocin)
3. Furosemide (Lasix)
4. Propranolol hydrochloride (Inderal)

30. The nurse is assessing a client 24 hours following


a cholecystectomy. The nurse notes that the T-tube has drained 750ml
of green-brown drainage. Which nursing intervention is most
appropriate?
1. Notify the physician
2. Document the findings
3. Irrigate the T-tube
4. Clamp the T-tube

31. The nurse provides medication instructions to a client with peptic


ulcer disease. Which statement, if made by the client, indicates the best
understanding of the medication therapy?

1. The cimetidine (Tagamet) will cause me to produce less stomach acid.


2. Sucralfate (Carafate) will change the fluid in my stomach.
3. Antacids will coat my stomach.
4. Omeprazole (Prilosec) will coat the ulcer and help it heal.

32. The client with peptic ulcer disease is scheduled for a pyloroplasty.
The client asks the nurse about the procedure. The nurse plans to
respond knowing that a pyloroplasty involves:

1. Cutting the vagus nerve


2. Removing the distal portion of the stomach
3. Removal of the ulcer and a large portion of the cells that produce
hydrochloric acid
4. An incision and resuturing of the pylorus to relax the muscle and enlarge the
opening from the stomach to the duodenum.

33. A client with a peptic ulcer is scheduled for a vagotomy. The client
asks the nurse about the purpose of this procedure. The nurse tells the
client that the procedure:

1. Decreases food absorption in the stomach


2. Heals the gastric mucosa
3. Halts stress reactions
4. Reduces the stimulus to acid secretions
34. The nurse would assess the client experiencing an acute episode
of cholecystitis for pain that is located in the right

1. Upper quadrant and radiates to the left scapula and shoulder


2. Upper quadrant and radiates to the right scapula and shoulder
3. Lower quadrant and radiates to the umbilicus
4. Lower quadrant and radiates to the back

35. Which of the following tasks should be included in the immediate


postoperative management of a client who has undergone gastric
resection?

1. Monitoring gastric pH to detect complications


2. Assessing for bowel sounds
3. Providing nutritional support
4. Monitoring for symptoms of hemorrhage

36. If a gastric acid perforates, which of the following actions should


not be included in the immediate management of the client?

1. Blood replacement
2. Antacid administration
3. Nasogastric tube suction
4. Fluid and electrolyte replacement

37. Mucosal barrier fortifiers are used in peptic ulcer disease


management for which of the following indications?

1. To inhibit mucus production


2. To neutralize acid production
3. To stimulate mucus production
4. To stimulate hydrogen ion diffusion back into the mucosa
38. When counseling a client in ways to prevent cholecystitis, which of
the following guidelines is most important?

1. Eat a low-protein diet


2. Eat a low-fat, low-cholesterol diet
3. Limit exercise to 10 minutes/day
4. Keep weight proportionate to height

39. Which of the following symptoms best describes Murphys sign?

1. Periumbilical ecchymosis exists


2. On deep palpation and release, pain in elicited
3. On deep inspiration, pain is elicited and breathing stops
4. Abdominal muscles are tightened in anticipation of palpation

40. Which of the following tests is most commonly used to


diagnose cholecystitis?

1. Abdominal CT scan
2. Abdominal ultrasound
3. Barium swallow
4. Endoscopy

41. Which of the following factors should be the main focus of nursing
management for a client hospitalized for cholecystitis?

1. Administration of antibiotics
2. Assessment for complications
3. Preparation for lithotripsy
4. Preparation for surgery

42. A client being treated for chronic cholecystitis should be given


which of the following instructions?
1. Increase rest
2. Avoid antacids
3. Increase protein in diet
4. Use anticholinergics as prescribed

43. The client with a duodenal ulcer may exhibit which of the following
findings on assessment?

1. Hematemesis
2. Malnourishment
3. Melena
4. Pain with eating

44. The pain of a duodenal ulcer can be distinguished from that of a


gastric ulcer by which of the following characteristics?

1. Early satiety
2. Pain on eating
3. Dull upper epigastric pain
4. Pain on empty stomach

45. The client has orders for a nasogastric (NG) tube insertion. During
the procedure, instructions that will assist in the insertion would be:

1. Instruct the client to tilt his head back for insertion in the nostril, then flex
his neck for the final insertion
2. After insertion into the nostril, instruct the client to extend his neck
3. Introduce the tube with the clients head tilted back, then instruct him to
keep his head upright for final insertion
4. Instruct the client to hold his chin down, then back for insertion of the tube

46. The most important pathophysiologic factor contributing to the


formation of esophageal varices is:
1. Decreased prothrombin formation
2. Decreased albumin formation by the liver
3. Portal hypertension
4. Increased central venous pressure

47. The client being treated for esophageal varices has a Sengstaken-
Blakemore tube inserted to control the bleeding. The most important
assessment is for the nurse to:

1. Check that the hemostat is on the bedside


2. Monitor IV fluids for the shift
3. Regularly assess respiratory status
4. Check that the balloon is deflated on a regular basis

48. A female client complains of gnawing epigastric pain for a few


hours after meals. At times, when the pain is severe, vomiting occurs.
Specific tests are indicated to rule out:

1. Cancer of the stomach


2. Peptic ulcer disease
3. Chronic gastritis
4. Pylorospasm

49. When a client has peptic ulcer disease, the nurse would expect a
priority intervention to be:

1. Assisting in inserting a Miller-Abbott tube


2. Assisting in inserting an arterial pressure line
3. Inserting a nasogastric tube
4. Inserting an I.V.

50. A 40-year-old male client has been hospitalized with peptic ulcer
disease. He is being treated with a histamine receptor antagonist
(cimetidine), antacids, and diet. The nurse doing discharge planning
will teach him that the action of cimetidine is to:

1. Reduce gastric acid output


2. Protect the ulcer surface
3. Inhibit the production of hydrochloric acid (HCl)
4. Inhibit vagus nerve stimulation

Answers and Rationale

1. Answer: 4. Weakness of the diaphragmatic muscle

A hiatal hernia is caused by weakness of the diaphragmatic muscle and


increased intra-abdominalnot intrathoracicpressure. This weakness allows
the stomach to slide into the esophagus. The esophageal supports weaken, but
esophageal muscle weakness or increased esophageal muscle pressure isnt a
factor in hiatal hernia.

2. Answer: 1. Obesity

Obesity may cause increased abdominal pressure that pushes the lower portion
of the stomach into the thorax.

3. Answer: 3. Esophageal reflux

Esophageal reflux is a common symptom of hiatal hernia. This seems to be


associated with chronic exposure of the lower esophageal sphincter to the lower
pressure of the thorax, making it less effective.

4. Answer: 3. Barium swallow


A barium swallow with fluoroscopy shows the position of the stomach in relation
to the diaphragm. A colonoscopy and a lower GI series show disorders of the
intestine.

5. Answer: 1. Recognizing hemorrhage

Recognizing the rupture of esophageal varices, or hemorrhage, is the focus of


nursing care because the client could succumb to this quickly. Controlling blood
pressure is also important because it helps reduce the risk of variceal rupture. It
is also important to teach the client what varices are and what foods he should
avoid such as spicy foods.

6. Answer: 4. Esophagogastroduodenoscopy (EGD)

The EGD can visualize the entire upper GI tract as well as allow for tissue
specimens and electrocautery if needed. The barium swallow could locate a
gastric ulcer. A CT scan and an abdominal x-ray arent useful in the diagnosis of
an ulcer.

7. Answer: 2. Reduce acid secretions

Ranitidine is a histamine-2 receptor antagonist that reduces acid secretion by


inhibiting gastrin secretion.

8. Answer: 3. On the left side with the head of the bed elevated 30
degrees

The discomfort of reflux is aggravated by positions that compress the abdomen


and the stomach. These include lying flat on the back or on the stomach after a
meal of lying on the right side. The left side-lying position with the head of the
bed elevated is most likely to give relief to the client.

9. Answer: 1. Irrigating the nasogastric tube


In a Billroth II procedure the proximal remnant of the stomach is anastomosed
to the proximal jejunum. Patency of the NG tube is critical for preventing the
retention of gastric secretions. The nurse should never irrigate or reposition the
gastric tube after gastric surgery, unless specifically ordered by the physician.
In this situation, the nurse would clarify the order.

10. Answer: 2. Limit the fluids taken with meals

The nurse should instruct the client to decrease the amount of fluid taken at
meals and to avoid high carbohydrate foods including fluids such as fruit
nectars; to assume a low-Fowlers position during meals; to lie down for 30
minutes after eating to delay gastric emptying; and to take antispasmodics as
prescribed.

11. Answer: 2. Use a soft toothbrush to brush the clients teeth after
each meal

A soft toothbrush should be used to brush the clients teeth after each meal and
more often as needed. Mechanical cleaning is necessary to maintain oral health,
simulate gingiva, and remove plaque. Assessing the oral cavity and recording
observations is the responsibility of the nurse, not the nursing assistant.
Swabbing with a safe foam applicator does not provide enough friction to clean
the mouth. Mouthwash can be a drying irritant and is not recommended for
frequent use.

12. Answer: 3. The client experiences a sudden increase in temperature

The most likely complication of an endoscopic procedure is perforation. A


sudden temperature spike with 1 to 2 hours after the procedure is indicative of
a perforation and should be reported immediately to the physician. A sore
throat is to be anticipated after an endoscopy. Clients are given sedatives during
the procedure, so it is expected that they will display signs of sedation after the
procedure is completed. A lack of appetite could be the result of many factors,
including the disease process.

13. Answer: 2. The procedure will result in anastomosis of the gastric


stump to the jejunum

A Billroth II procedure bypasses the duodenum and connects the gastric stump
directly to the jejunum. The pyloric sphincter is removed, along with some of
the stomach fundus.

14. Answer: 1. Dark brown

About 12 to 24 hours after a subtotal gastrectomy, gastric drainage is normally


brown, which indicates digested blood. Bile green or cloudy white drainage is
not expected during the first 12 to 24 hours after a subtotal gastrectomy.
Drainage during the first 6 to 12 hours contains some bright red blood, but
large amounts of blood or excessively bloody drainage should be reported to the
physician promptly.

15. Answer: 3. Monitor the client for N/V, and abdominal distention

Nausea, vomiting, or abdominal distention indicated that gas and secretions are
accumulating within the gastric pouch due to impaired peristalsis or edema at
the operative site and may indicate that the drainage system is not working
properly. Saline solution is used to irrigate nasogastric tubes. Hypotonic
solutions such as water increase electrolyte loss. In addition, a physicians order
is needed to irrigate the NG tube, because this procedure could disrupt the
suture line. After gastric surgery, only the surgeon repositions the NG
tube because of the danger of rupturing or dislodging the suture line. The
amount of suction varies with the type of tube used and is ordered by the
physician. High suction may create too much tension on the gastric suture line.
16. Answer: 4. Achieve optimal nutritional status through oral or
parenteral feedings

An appropriate expected outcome is for the client to achieve optimal nutritional


status through the use of oral feedings or total parenteral nutrition (TPN). TPN
may be used to supplement oral intake, or it may be used alone if the client
cannot tolerate oral feedings. The client would not be expected to regain lost
weight within 1 month after surgery or to tolerate a normal dietary intake of
three meals per day. Nausea and vomiting would not be considered an expected
outcome of gastric surgery, and regular use of antiemetics would not be
anticipated.

17. Answer: 4. Aspiration of gastric contents

Clients with GERD can develop pulmonary symptoms such as coughing,


wheezing, and dyspnea that are caused by the aspiration of gastric contents.
GERD does not predispose the client to the development of laryngeal cancer.
Irritation of the esophagus and esophageal scar tissue formation can develop as
a result of GERD. However, GERD is more likely to cause painful and difficult
swallowing.

18. Answer: 1. Eating small, frequent meals

Esophageal reflux worsens when the stomach is overdistended with food.


Therefore, an important measure is to eat small, frequent meals. Fluid intake
should be decreased during meals to reduce abdominal distention. Avoiding air
swallowing does not prevent esophageal reflux. Food intake in the evening
should be strictly limited to reduce the incidence of nighttime reflux, so bedtime
snacks are not recommended.

19. Answer: 4. The ulcer has perforated


The body reacts to perforation of an ulcer by immobilizing the area as much as
possible. This results in boardlike muscle rigidity, usually with extreme pain.
Perforation is a medical emergency requiring immediate surgical intervention
because peritonitis develops quickly after perforation. An intestinal obstruction
would not cause midepigastric pain. Esophageal inflammation or the
development of additional ulcers would not cause a rigid, boardlike abdomen.

20. Answers: 3 and 4.

Vomiting and weight loss are common with gastric ulcers. Clients with a gastric
ulcer are most likely to complain of a burning epigastric pain that occurs about
one hour after eating. Eating frequently aggravates the pain. Clients with
duodenal ulcers are more likely to complain about pain that occurs during the
night and is frequently relieved by eating.

21. Answers: 2, 4, and 5.

Following a gastroscopy, the nurse should monitor the client for complications,
which include perforation and the potential for aspiration. An elevated
temperature, complaints of epigastric pain, or the vomiting of blood
(hematemesis) are all indications of a possible perforation and should be
reported promptly. A sore throat is a common occurrence following a
gastroscopy. Clients are usually sedated to decrease anxiety and the nurse
would anticipate that the client will be drowsy following the procedure.

22. Answer: 2. Deficient knowledge related to unfamiliarity with


significant signs and symptoms

Black, tarry stools are an important warning sign of bleeding in peptic ulcer
disease. Digested blood in the stomach causes it to be black. The odor of the
stool is very stinky. Clients with peptic ulcer disease should be instructed to
report the incidence of black stools promptly to their physician.
23. Answer: 2. Disturbed Sleep Pattern related to epigastric pain

Based on the data provided, the most appropriate nursing diagnosis would be
Disturbed Sleep pattern. A client with a duodenal ulcer commonly awakens at
night with pain. The clients feelings of anxiety do not necessarily indicate that
she is coping ineffectively.

24. Answers: 2 and 3.

The symptoms of nausea and dizziness in a client with peptic ulcer disease may
be indicative of hemorrhage and should not be ignored. The appropriate nursing
actions at this time are for the nurse to monitor the clients vital signs and
notify the physician of the clients symptoms. To administer an antacid hourly or
to wait one hour to reassess the client would be inappropriate; prompt
intervention is essential in a client who is potentially experiencing a
gastrointestinal hemorrhage. The nurse would notify the physician of
assessment findings and then initiate oxygen therapy if ordered by the
physician.

25. Answer: 3. At bedtime

Ranitidine blocks secretion of hydrochloric acid. Clients who take only one daily
dose of ranitidine are usually advised to take it at bedtime to inhibit nocturnal
secretion of acid. Clients who take the drug twice a day are advised to take it in
the morning and at bedtime.

26. Answer: 3. The client is experiencing a side effect of the aluminum


hydroxide.

It is most likely that the client is experiencing a side effect of the antacid.
Antacids with aluminum salt products, such as aluminum hydroxide, form
insoluble salts in the body. These precipitate and accumulate in the intestines,
causing constipation. Increasing dietary fiber intake or daily exercise may be a
beneficial lifestyle change for the client but is not likely to
relieve constipation caused by the aluminum hydroxide. Constipation, in
isolation from other symptoms, is not a sign of bowel obstruction.

27. Answer: 4. It is best for me to take my antacid 1 to 3 hours after


meals.

Antacids are most effective if taken 1 to 3 hours after meals and at bedtime.
When an antacid is taken on an empty stomach, the duration of the drugs
action is greatly decreased. Taking antacids 1 to 3 hours after a meal lengthens
the duration of action, thus increasing the therapeutic action of the drug.
Antacids should be administered about 2 hours after other medications to
decrease the chance of drug interactions. It is not necessary to decrease fluid
intake when taking antacids.

28. Answer: 2. Vitamin B12

Chronic gastritis causes deterioration and atrophy of the lining of the stomach,
leading to the loss of the functioning parietal cells. The source of the intrinsic
factor is lost, which results in the inability to absorb vitamin B12. This leads to
the development of pernicious anemia.

29. Answer: 2. Indomethacin (Indocin)

Indomethacin (Indocin) is a NSAID and can cause ulceration of the esophagus,


stomach, duodenum, or small intestine. Indomethacin is contraindicated in a
client with GI disorders.

30. Answer: 2. Document the findings

Following cholecystectomy, drainage from the T-tube is initially bloody and then
turns to green-brown. The drainage is measured as output. The amount of
expected drainage will range from 500 to 1000 ml per day. The nurse would
document the output.

31. Answer: 1 The cimetidine (Tagamet) will cause me to produce less


stomach acid.

Cimetidine (Tagamet), a histamine H2 receptor antagonist, will decrease the


secretion of gastric acid. Sucralfate (Carafate) promotes healing by coating the
ulcer. Antacids neutralize acid in the stomach. Omeprazole (Prilosec) inhibits
gastric acid secretion.

32. Answer: 4. An incision and resuturing of the pylorus to relax the


muscle and enlarge the opening from the stomach to the duodenum.

Option 4 describes the procedure for a pyloroplasty. A vagotomy involves


cutting the vagus nerve. A subtotal gastrectomy involves removing the distal
portion of the stomach. A Billroth II procedure involves removal of the ulcer and
a large portion of the tissue that produces hydrochloric acid.

33. Answer: 4. Reduces the stimulus to acid secretions

A vagotomy, or cutting the vagus nerve, is done to eliminate parasympathetic


stimulation of gastric secretion.

34. Answer: 2. Upper quadrant and radiates to the right scapula and
shoulder

During an acute gallbladder attack, the client may complain of severe right
upper quadrant pain that radiates to the right scapula and shoulder. This is
governed by the pattern on dermatomes in the body.

35. Answer: 4. Monitoring for symptoms of hemorrhage


The client should be monitored closely for signs and symptoms of hemorrhage,
such as bright red blood in the nasogastric tube suction, tachycardia, or a drop
in blood pressure. Gastric pH may be monitored to evaluate the need for
histamine-2 receptor antagonists. Bowel sounds may not return for up to 72
hours postoperatively. Nutritional needs should be addressed soon
after surgery.

36. Answer: 2. Antacid administration

Antacids arent helpful in perforation. The client should be treated


with antibiotics as well as fluid, electrolyte, and blood replacement. NG
tube suction should also be performed to prevent further spillage of stomach
contents into the peritoneal cavity.

37. Answer: 3. To stimulate mucus production

The mucosal barrier fortifiers stimulate mucus production and prevent hydrogen
ion diffusion back into the mucosa, resulting in accelerated ulcer healing.
Antacids neutralize acid production.

38. Answer: 4. Keep weight proportionate to height

Obesity is a known cause of gallstones, and maintaining a recommended weight


will help protect against gallstones. Excessive dietary intake of cholesterol is
associated with the development of gallstones in many people. Dietary protein
isnt implicated in cholecystitis. Liquid protein and low-calorie diets (with rapid
weight loss of more than 5 lb [2.3kg] per week) are implicated as the cause of
some cases of cholecystitis. Regular exercise (30 minutes/three times a week)
may help reduce weight and improve fat metabolism. Reducing stress may
reduce bile production, which may also indirectly decrease the chances of
developing cholecystitis.

39. Answer: 3. On deep inspiration, pain is elicited and breathing stops


Murphys sign is elicited when the client reacts to pain and stops breathing. Its
a common finding in clients with cholecystitis. Periumbilical ecchymosis, Cullens
sign, is present in peritonitis. Pain on deep palpation and release is rebound
tenderness. Tightening up abdominal muscles in anticipation of palpation is
guarding.

40. Answer: 2. Abdominal ultrasound

An abdominal ultrasound can show if the gallbladder is enlarged, if gallstones


are present, if the gallbladder wall is thickened, or if distention of the
gallbladder lumen is present. An abdominal CT scan can be used to diagnose
cholecystitis, but it usually isnt necessary. A barium swallow looks at the
stomach and the duodenum. Endoscopy looks at the esophagus, stomach, and
duodenum.

41. Answer: 2. Assessment for complications

The client with acute cholecystitis should first be monitored for


perforation, fever, abscess, fistula, and sepsis. After assessment, antibiotics will
be administered to reduce the infection. Lithotripsy is used only for a small
percentage of clients. Surgery is usually done after the acute infection has
subsided.

42. Answer: 4. Use anticholinergics as prescribed

Conservative therapy for chronic cholecystitis includes weight reduction by


increasing physical activity, a low-fat diet, antacid use to treat dyspepsia, and
anticholinergic use to relax smooth muscles and reduce ductal tone and spasm,
thereby reducing pain.

43. Answer: 3. Melena


The client with a duodenal ulcer may have bleeding at the ulcer site, which
shows up as melena (black tarry stool). The other findings are consistent with a
gastric ulcer.

44. Answer: 4. Pain on empty stomach

Pain on empty stomach is relieved by taking foods or antacids. The other


symptoms are those of a gastric ulcer.

45. Answer: 1. Instruct the client to tilt his head back for insertion in
the nostril, then flex his neck for the final insertion

NG insertion technique is to have the client first tilt his head back for insertion
into the nostril, then to flex his neck forward and swallow. Extension of the neck
(2) will impede NG tube insertion.

46. Answer: 3. Portal hypertension

As the liver cells become fatty and degenerate, they are no longer able to
accommodate a large amount of blood necessary for homeostasis. The pressure
in the liver increases and causes increased pressure in the venous system. As
the portal pressure increases, fluid exudes into the abdominal cavity. This is
called ascites.

47. Answer: 3. Regularly assess respiratory status

The respiratory system can become occluded if the balloon slips and moves up
the esophagus, putting pressure on the trachea. This would result in respiratory
distress and should be assessed frequently. Scissors should be kept at the
bedside to cut the tube if distress occurs. This is a safety intervention.

48. Answer: 2. Peptic ulcer disease


Peptic ulcer disease is characteristically gnawing epigastric pain that may
radiate to the back. Vomiting usually reflects pyloric spasm from muscular
spasm or obstruction. Cancer (1) would not evidence pain or vomiting unless
the pylorus was obstructed.

49. Answer: 3. Inserting a nasogastric tube

An NG tube insertion is the most appropriate intervention because it will


determine the presence of active GI bleeding. A Miller-Abbott tube (1) is a
weighted, mercury-filled ballooned tube used to resolve bowel obstructions.
There is no evidence of shock or fluid overload in the client; therefore, an
arterial line (2) is not appropriate at this time and an IV (4) is optional.

50. Answer: 1. Reduce gastric acid output

These drugs inhibit the action of histamine on the H2 receptors of parietal cells,
thus reducing gastric acid output.

1. Nurse Berlinda is assigned to a 41-year-old client who has a


diagnosis of chronic pancreatitis. The nurse reviews the laboratory
result, anticipating a laboratory report that indicates a serum amylase
level of:

A. 45 units/L
B. 100 units/L
C. 300 units/L
D. 500 units/L

2. A male client who is recovering from surgery has been advanced


from a clear liquid diet to a full liquid diet. The client is looking forward
to the diet change because he has been bored with the clear liquid
diet. The nurse would offer which full liquid item to the client?
A. Tea
B. Gelatin
C. Custard
D. Popsicle

3. Nurse Juvy is caring for a client with cirrhosis of the liver. To


minimize the effects of the disorder, the nurse teaches the client about
foods that are high in thiamine. The nurse determines that the client
has the best understanding of the dietary measures to follow if the
client states an intention to increase the intake of:

A. Pork
B. Milk
C. Chicken
D. Broccoli

4. Nurse Oliver checks for residual before administering a bolus tube


feeding to a client with a nasogastric tube and obtains a residual
amount of 150 mL. What is appropriate action for the nurse to take?

A. Hold the feeding


B. Reinstill the amount and continue with administering the feeding
C. Elevate the clients head at least 45 degrees and administer the feeding
D. Discard the residual amount and proceed with administering the feeding

5. A nurse is inserting a nasogastric tube in an adult male client. During


the procedure, the client begins to cough and has difficulty breathing.
Which of the following is the appropriate nursing action?

A. Quickly insert the tube


B. Notify the physician immediately
C. Remove the tube and reinsert when the respiratory distress subsides
D. Pull back on the tube and wait until the respiratory distress subsides
6. Nurse Ryan is assessing for correct placement of a nasogastric tube.
The nurse aspirates the stomach contents and checks the contents for
pH. The nurse verifies correct tube placement if which pH value is
noted?

A. 3.5
B. 7.0
C. 7.35
D. 7.5

7. A nurse is preparing to remove a nasogastric tube from a female


client. The nurse should instruct the client to do which of the following
just before the nurse removes the tube?

A. Exhale
B. Inhale and exhale quickly
C. Take and hold a deep breath
D. Perform a Valsalva maneuver

8. Nurse Joy is preparing to administer medication through


a nasogastric tube that is connected to suction. To administer the
medication, the nurse would:

A. Position the client supine to assist in medication absorption


B. Aspirate the nasogastric tube after medication administration to maintain
patency
C. Clamp the nasogastric tube for 30 minutes following administration of the
medication
D. Change the suction setting to low intermittent suction for 30 minutes after
medication administration

9. A nurse is preparing to care for a female client with esophageal


varices who just had a Sengstaken-Blakemore tube inserted. The nurse
gathers supplies, knowing that which of the following items must be
kept at the bedside at all times?

A. An obturator
B. Kelly clamp
C. An irrigation set
D. A pair of scissors

10. Dr. Smith has determined that the client with hepatitis has
contracted the infection from contaminated food. The nurse
understands that this client is most likely experiencing what type
of hepatitis?

A. Hepatitis A
B. Hepatitis B
C. Hepatitis C
D. Hepatitis D

11. A client is suspected of having hepatitis. Which diagnostic test


result will assist in confirming this diagnosis?

A. Elevated hemoglobin level


B. Elevated serum bilirubin level
C. Elevated blood urea nitrogen level
D. Decreased erythrocyte sedimentation rate

12. The nurse is reviewing the physicians orders written for a male
client admitted to the hospital with acute pancreatitis. Which physician
order should the nurse question if noted on the clients chart?

A. NPO status
B. Nasogastric tube inserted
C. Morphine sulfate for pain
D. An anticholinergic medication
13. A female client being seen in a physicians office has just been
scheduled for a barium swallow the next day. The nurse writes down
which instruction for the client to follow before the test?

A. Fast for 8 hours before the test


B. Eat a regular supper and breakfast
C. Continue to take all oral medications as scheduled
D. Monitor own bowel movement pattern for constipation

14. The nurse is performing an abdominal assessment and inspects the


skin of the abdomen. The nurse performs which assessment technique
next?

A. Palpates the abdomen for size


B. Palpates the liver at the right rib margin
C. Listens to bowel sounds in all for quadrants
D. Percusses the right lower abdominal quadrant

15. Polyethylene glycol-electrolyte solution (GoLYTELY) is prescribed


for the female client scheduled for a colonoscopy. The client begins to
experience diarrhea following administration of the solution. What
action by the nurse is appropriate?

A. Start an IV infusion
B. Administer an enema
C. Cancel the diagnostic test
D. Explain that diarrhea is expected

16. The nurse is caring for a male client with a diagnosis of chronic
gastritis. The nurse monitors the client knowing that this client is at
risk for which vitamin deficiency?

A. Vitamin A
B. Vitamin B12
C. Vitamin C
D. Vitamin E

17. The nurse is reviewing the medication record of a female client with
acute gastritis. Which medication, if noted on the clients record, would
the nurse question?

A. Digoxin (Lanoxin)
B. Furosemide (Lasix)
C. Indomethacin (Indocin)
D. Propranolol hydrochloride (Inderal)

18. The nurse is assessing a male client 24 hours following


a cholecystectomy. The nurse noted that the T-tube has drained 750 mL
of green-brown drainage since the surgery. Which nursing intervention
is appropriate?

A. Clamp the T-tube


B. Irrigate the T-tube
C. Notify the physician
D. Document the findings

19. The nurse is monitoring a female client with a diagnosis of peptic


ulcer. Which assessment findings would most likely indicate perforation
of the ulcer?

A. Bradycardia
B. Numbness in the legs
C. Nausea and vomiting
D. A rigid, board-like abdomen

20. A male client with a peptic ulcer is scheduled for a vagotomy and
the client asks the nurse about the purpose of this procedure. Which
response by the nurse best describes the purpose of a vagotomy?
A. Halts stress reactions
B. Heals the gastric mucosa
C. Reduces the stimulus to acid secretions
D. Decreases food absorption in the stomach

21. The nurse is caring for a female client following a Billroth II


procedure. Which postoperative order should the nurse question and
verify?

A. Leg exercises
B. Early ambulation
C. Irrigating the nasogastric tube
D. Coughing and deep-breathing exercises

22. The nurse is providing discharge instructions to a male client


following gastrectomy and instructs the client to take which measure to
assist in preventing dumping syndrome?

A. Ambulate following a meal


B. Eat high carbohydrate foods
C. Limit the fluid taken with meal
D. Sit in a high-Fowlers position during meals

23. The nurse is monitoring a female client for the early signs and
symptoms of dumping syndrome. Which of the following indicate this
occurrence?

A. Sweating and pallor


B. Bradycardia and indigestion
C. Double vision and chest pain
D. Abdominal cramping and pain

24. The nurse is preparing a discharge teaching plan for the male client
who had umbilical hernia repair. What should the nurse include in the
plan?
A. Irrigating the drain
B. Avoiding coughing
C. Maintaining bed rest
D. Restricting pain medication

25. The nurse is instructing the male client who has an inguinal hernia
repair how to reduce postoperative swelling following the procedure.
What should the nurse tell the client?

A. Limit oral fluid


B. Elevate the scrotum
C. Apply heat to the abdomen
D. Remain in a low-fiber diet

26. The nurse is caring for a hospitalized female client with a diagnosis
of ulcerative colitis. Which finding, if noted on assessment of the client,
would the nurse report to the physician?

A. Hypotension
B. Bloody diarrhea
C. Rebound tenderness
D. A hemoglobin level of 12 mg/dL

27. The nurse is caring for a male client postoperatively following


creation of a colostomy. Which nursing diagnosis should the nurse
include in the plan of care?

A. Sexual dysfunction
B. Body image, disturbed
C. Fear related to poor prognosis
D. Nutrition: more than body requirements, imbalanced

28. The nurse is reviewing the record of a female client with Crohns
disease. Which stool characteristics should the nurse expect to note
documented in the clients record?
A. Diarrhea
B. Chronic constipation
C. Constipation alternating with diarrhea
D. Stools constantly oozing from the rectum

29. The nurse is performing a colostomy irrigation on a male client.


During the irrigation, the client begins to complain of abdominal
cramps. What is the appropriate nursing action?

A. Notify the physician


B. Stop the irrigation temporarily
C. Increase the height of the irrigation
D. Medicate for pain and resume the irrigation

30. The nurse is teaching a female client how to perform a colostomy


irrigation. To enhance the effectiveness of the irrigation and fecal
returns, what measure should the nurse instruct the client to do?

A. Increase fluid intake


B. Place heat on the abdomen
C. Perform the irrigation in the evening
D. Reduce the amount of irrigation solution

Answers and Rationale

1. Answer: C. 300 units/L

The normal serum amylase level is 25 to 151 units/L. With chronic cases
of pancreatitis, the rise in serum amylase levels usually does not exceed three
times the normal value. In acute pancreatitis, the value may exceed five times
the normal value. Options A and B are within normal limits. Option D is an
extremely elevated level seen in acute pancreatitis.

2. Answer: C. Custard
Full liquid food items include items such as plain ice cream, sherbet, breakfast
drinks, milk, pudding and custard, soups that are strained, and strained
vegetable juices. A clear liquid diet consists of foods that are relatively
transparent. The food items in options A, B, and D are clear liquids.

3. Answer: A. Pork

The client with cirrhosis needs to consume foods high in thiamine. Thiamine is
present in a variety of foods of plant and animal origin. Pork products are
especially rich in this vitamin. Other good food sources include nuts, whole grain
cereals, and legumes. Milk contains vitamins A, D, and B2. Poultry contains
niacin. Broccoli contains vitamins C, E, and K and folic acid

4. Answer: A. Hold the feeding

Unless specifically indicated, residual amounts more than 100 mL require


holding the feeding. Therefore options B, C, and D are incorrect. Additionally,
the feeding is not discarded unless its contents are abnormal in color or
characteristics.

5. Answer: D. Pull back on the tube and wait until the respiratory
distress subsides

During the insertion of a nasogastric tube, if the client experiences difficulty


breathing or any respiratory distress, withdraw the tube slightly, stop the tube
advancement, and wait until the distress subsides. Options B and C are
unnecessary. Quickly inserting the tube is not an appropriate action because, in
this situation, it may be likely that the tube has entered the bronchus.

6. Answer: A. 3.5

If the nasogastric tube is in the stomach, the pH of the contents will be acidic.
Gastric aspirates have acidic pH values and should be 3.5 or lower. Option B
indicates a slightly acidic pH. Option C indicates a neutral pH. Option D indicates
an alkaline pH.
7. Answer: C. Take and hold a deep breath

When the nurse removes a nasogastric tube, the client is instructed to take and
hold a deep breath. This will close the epiglottis. This allows for easy withdrawal
through the esophagus into the nose. The nurse removes the tube with one
smooth, continuous pull.

8. Answer: C. Clamp the nasogastric tube for 30 minutes following


administration of the medication

If a client has a nasogastric tube connected to suction, the nurse should wait up
to 30 minutes before reconnecting the tube to the suction apparatus to allow
adequate time for medication absorption. Aspirating the nasogastric tube will
remove the medication just administered. Low intermittent suction also will
remove the medication just administered. The client should not be placed in
the supine position because of the risk for aspiration.

9. Answer: D. A pair of scissors

When the client has a Sengstaken-Blakemore tube, a pair of scissors must be


kept at the clients bedside at all times. The client needs to be observed for
sudden respiratory distress, which occurs if the gastric balloon ruptures and the
entire tube moves upward. If this occurs, the nurse immediately cuts all balloon
lumens and removes the tube. An obturator and a Kelly clamp are kept at the
bedside of a client with a tracheostomy. An irrigation set may be kept at the
bedside, but it is not the priority item.

10. Answer: A. Hepatitis A

Hepatitis A is transmitted by the fecal-oral route via contaminated food or


infected food handlers. Hepatitis B, C, and D are transmitted most commonly
via infected blood or body fluids.

11. Answer: B. Elevated serum bilirubin level


Laboratory indicators of hepatitis include elevated liver enzyme levels, elevated
serum bilirubin levels, elevated erythrocyte sedimentation rates, and
leukopenia. An elevated blood urea nitrogen level may indicate renal
dysfunction. A hemoglobin level is unrelated to this diagnosis.

12. Answer: C. Morphine sulfate for pain

Meperidine (Demerol) rather than morphine sulfate is the medication of choice


to treat pain because morphine sulfate can cause spasms in the sphincter of
Oddi. Options A, B, and D are appropriate interventions for the client with
acute pancreatitis.

13. Answer: A. Fast for 8 hours before the test

A barium swallow is an x-ray study that uses a substance called barium for
contrast to highlight abnormalities in the gastrointestinal tract. The client should
fast for 8 to 12 hours before the test, depending on physician instructions. Most
oral medications also are withheld before the test. After the procedure, the
nurse must monitor for constipation, which can occur as a result of the presence
of barium in the gastrointestinal tract.

14. Answer: C. Listens to bowel sounds in all for quadrants

The appropriate sequence for abdominal examination is inspection, auscultation,


percussion, and palpation. Auscultation is performed after inspection to ensure
that the motility of the bowel and bowel sounds are not altered by percussion or
palpation. Therefore, after inspecting the skin on the abdomen, the nurse
should listen for bowel sounds.

15. Answer: D. Explain that diarrhea is expected

The solution GoLYTELY is a bowel evacuant used to prepare a client for a


colonoscopy by cleansing the bowel. The solution is expected to cause a mild
diarrhea and will clear the bowel in 4 to 5 hours. Options A, B, and C are
inappropriate actions.
16. Answer: B. Vitamin B12

Chronic gastritis causes deterioration and atrophy of the lining of the stomach,
leading to the loss of the function of the parietal cells. The source of the intrinsic
factor is lost, which results in the inability to absorb vitamin B12. This leads to
the development of pernicious anemia. The client is not at risk for vitamin A, C,
or E deficiency.

17. Answer: C. Indomethacin (Indocin)

Indomethacin (Indocin) is a nonsteroidal anti-inflammatory drug and can cause


ulceration of the esophagus, stomach, or small intestine. Indomethacin is
contraindicated in a client with gastrointestinal disorders. Furosemide (Lasix) is
a loop diuretic. Digoxin is a cardiac medication. Propranolol (Inderal) is a -
adrenergic blocker. Furosemide, digoxin, and propranolol are not contraindicated
in clients with gastric disorders.

18. Answer: D. Document the findings

Following cholecystectomy, drainage from the T-tube is initially bloody and then
turns to a greenish-brown color. The drainage is measured as output. The
amount of expected drainage will range from 500 to 1000 mL/day. The nurse
would document the output.

19. Answer: D. A rigid, board-like abdomen

Perforation of an ulcer is a surgical emergency and is characterized by sudden,


sharp, intolerable severe pain beginning in the mid epigastric area and
spreading over the abdomen, which becomes rigid and board-like. Nausea and
vomiting may occur. Tachycardia may occur as hypovolemic shock develops.
Numbness in the legs is not an associated finding.

20. Answer: C. Reduces the stimulus to acid secretions


A vagotomy, or cutting of the vagus nerve, is done to eliminate parasympathetic
stimulation of gastric secretion. Options A, B, and D are incorrect descriptions of
a vagotomy.

21. Answer: C. Irrigating the nasogastric tube

In a Billroth II procedure, the proximal remnant of the stomach is anastomosed


to the proximal jejunum. Patency of the nasogastric tube is critical for
preventing the retention of gastric secretions. The nurse should never irrigate or
reposition the gastric tube after gastric surgery, unless specifically ordered by
the physician. In this situation, the nurse should clarify the order. Options A, B,
and D are appropriate postoperative interventions.

22. Answer: C. Limit the fluid taken with meal

Dumping syndrome is a term that refers to a constellation of vasomotor


symptoms that occurs after eating, especially following a Billroth II procedure.
Early manifestations usually occur within 30 minutes of eating and include
vertigo, tachycardia, syncope, sweating, pallor, palpitations, and the desire to lie
down. The nurse should instruct the client to decrease the amount of fluid taken
at meals and to avoid high-carbohydrate foods, including fluids such as fruit
nectars; to assume a low-Fowlers position during meals; to lie down for 30
minutes after eating to delay gastric emptying; and to take antispasmodics as
prescribed.

23. Answer: A. Sweating and pallor

Early manifestations of dumping syndrome occur 5 to 30 minutes after eating.


Symptoms include vertigo, tachycardia, syncope, sweating, pallor, palpitations,
and the desire to lie down.

24. Answer: B. Avoiding coughing

Coughing is avoided following umbilical hernia repair to prevent disruption of


tissue integrity, which can occur because of the location of this surgical
procedure. Bed rest is not required following this surgical procedure. The client
should take analgesics as needed and as prescribed to control pain. A drain is
not used in this surgical procedure, although the client may be instructed in
simple dressing changes.

25. Answer: B. Elevate the scrotum

Following inguinal hernia repair, the client should be instructed to elevate the
scrotum and apply ice packs while in bed to decrease pain and swelling. The
nurse also should instruct the client to apply a scrotal support when out of bed.
Heat will increase swelling. Limiting oral fluids and a low-fiber diet can
cause constipation.

26. Answer: C. Rebound tenderness

Rebound tenderness may indicate peritonitis. Bloody diarrhea is expected to


occur in ulcerative colitis. Because of the blood loss, the client may be
hypotensive and the hemoglobin level may be lower than normal. Signs
of peritonitis must be reported to the physician.

27. Answer: B. Body image, disturbed

Body image, disturbed relates to loss of bowel control, the presence of a stoma,
the release of fecal material onto the abdomen, the passage of flatus, odor, and
the need for an appliance (external pouch). No data in the question support
options A and C. Nutrition: less than body requirements, imbalanced is the
more likely nursing diagnosis.

28. Answer: A. Diarrhea

Crohns disease is characterized by nonbloody diarrhea of usually not more than


four to five stools daily. Over time, the diarrhea episodes increase in frequency,
duration, and severity. Options B, C, and D are not characteristics of Crohns
disease.
29. Answer: B. Stop the irrigation temporarily

If cramping occurs during a colostomy irrigation, the irrigation flow is stopped


temporarily and the client is allowed to rest. Cramping may occur from an
infusion that is too rapid or is causing too much pressure. The physician does
not need to be notified. Increasing the height of the irrigation will cause further
discomfort. Medicating the client for pain is not the appropriate action in this
situation.

30. Answer: A. Increase fluid intake

To enhance effectiveness of the irrigation and fecal returns, the client is


instructed to increase fluid intake and to take other measures to
prevent constipation. Options B, C and D will not enhance the effectiveness of
this procedure.

1. During preparation for bowel surgery, a male client receives


an antibiotic to reduce intestinal bacteria. Antibiotic therapy may
interfere with synthesis of which vitamin and may lead to
hypoprothrombinemia?

A. vitamin A
B. vitamin D
C. vitamin E
D. vitamin K

2. When evaluating a male client for complications of


acute pancreatitis, the nurse would observe for:

A. increased intracranial pressure.


B. decreased urine output.
C. bradycardia.
D. hypertension.
3. A male client with a recent history of rectal bleeding is being
prepared for a colonoscopy. How should the nurse position the client
for this test initially?

A. Lying on the right side with legs straight


B. Lying on the left side with knees bent
C. Prone with the torso elevated
D. Bent over with hands touching the floor

4. A male client with extreme weakness, pallor, weak peripheral pulses,


and disorientation is admitted to the emergency department. His wife
reports that he has been spitting up blood. A Mallory-Weiss tear is
suspected, and the nurse begins taking a client history from the clients
wife. The question by the nurse that demonstrates her understanding of
Mallory-Weiss tearing is:

A. Tell me about your husbands alcohol usage.


B. Is your husband being treated for tuberculosis?
C. Has your husband recently fallen or injured his chest?
D. Describe spices and condiments your husband uses on food.

5. Which of the following nursing interventions should the nurse


perform for a female client receiving enteral feedings through a
gastrostomy tube?

A. Change the tube feeding solutions and tubing at least every 24 hours.
B. Maintain the head of the bed at a 15-degree elevation continuously.
C. Check the gastrostomy tube for position every 2 days.
D. Maintain the client on bed rest during the feedings.

6. A male client is recovering from a small-bowel resection. To


relieve pain, the physician prescribes meperidine (Demerol), 75 mg I.M.
every 4 hours. How soon after administration should meperidine onset
of action occur?
A. 5 to 10 minutes
B. 15 to 30 minutes
C. 30 to 60 minutes
D. 2 to 4 hours

7. The nurse is caring for a male client with cirrhosis. Which


assessment findings indicate that the client has deficient vitamin K
absorption caused by this hepatic disease?

A. Dyspnea and fatigue


B. Ascites and orthopnea
C. Purpura and petechiae
D. Gynecomastia and testicular atrophy

8. Which condition is most likely to have a nursing diagnosis of fluid


volume deficit?

A. Appendicitis
B. Pancreatitis
C. Cholecystitis
D. Gastric ulcer

9. While a female client is being prepared for discharge, the nasogastric


(NG) feeding tube becomes cloggeD. To remedy this problem and teach
the clients family how to deal with it at home, what should the nurse
do?

A. Irrigate the tube with cola.


B. Advance the tube into the intestine.
C. Apply intermittent suction to the tube.
D. Withdraw the obstruction with a 30-ml syringe.

10. A male client with pancreatitis complains of pain. The nurse expects
the physician to prescribe meperidine (Demerol) instead of morphine to
relieve pain because:
A. meperidine provides a better, more prolonged analgesic effect.
B. morphine may cause spasms of Oddis sphincter.
C. meperidine is less addictive than morphine.
D. morphine may cause hepatic dysfunction.

11. Mandy, an adolescent girl is admitted to an acute care facility with


severe malnutrition. After a thorough examination, the physician
diagnoses anorexia nervosa. When developing the plan of care for this
client, the nurse is most likely to include which nursing diagnosis?

A. Hopelessness
B. Powerlessness
C. Chronic low self-esteem
D. Deficient knowledge

12. Which diagnostic test would be used first to evaluate a client with
upper GI bleeding?

A. Endoscopy
B. Upper GI series
C. Hemoglobin (Hb) levels and hematocrit (HCT)
D. Arteriography

13. A female client who has just been diagnosed with hepatitis A asks,
How could I have gotten this disease? What is the nurses best
response?

A. You may have eaten contaminated restaurant food.


B. You could have gotten it by using I.V. drugs.
C. You must have received an infected blood transfusion.
D. You probably got it by engaging in unprotected sex.

14. When preparing a male client, age 51, for surgery to


treat appendicitis, the nurse formulates a nursing diagnosis of Risk for
infection related to inflammation, perforation, and surgery. What is the
rationale for choosing this nursing diagnosis?

A. Obstruction of the appendix may increase venous drainage and cause the
appendix to rupture.
B. Obstruction of the appendix reduces arterial flow, leading to ischemia,
inflammation, and rupture of the appendix.
C. The appendix may develop gangrene and rupture, especially in a middle-
aged client.
D. Infection of the appendix diminishes necrotic arterial blood flow and
increases venous drainage.

15. A female client with hepatitis C develops liver failure and GI


hemorrhage. The blood products that would most likely bring about
hemostasis in the client are:

A. whole blood and albumin.


B. platelets and packed red blood cells.
C. fresh frozen plasma and whole blood.
D. cryoprecipitate and fresh frozen plasma.

16. To prevent gastroesophageal reflux in a male client with hiatal


hernia, the nurse should provide which discharge instruction?

A. Lie down after meals to promote digestion.


B. Avoid coffee and alcoholic beverages.
C. Take antacids with meals.
D. Limit fluid intake with meals.

17. The nurse caring for a client with small-bowel obstruction would
plan to implement which nursing intervention first?

A. Administering pain medication


B. Obtaining a blood sample for laboratory studies
C. Preparing to insert a nasogastric (NG) tube
D. Administering I.V. fluids

18. A female client with dysphagia is being prepared for discharge.


Which outcome indicates that the client is ready for discharge?

A. The client doesnt exhibit rectal tenesmus.


B. The client is free from esophagitis and achalasia.
C. The client reports diminished duodenal inflammation.
D. The client has normal gastric structures.

19. A male client undergoes total gastrectomy. Several hours after


surgery, the nurse notes that the clients nasogastric (NG) tube has
stopped draining. How should the nurse respond?

A. Notify the physician


B. Reposition the tube
C. Irrigate the tube
D. Increase the suction level

20. What laboratory finding is the primary diagnostic indicator


for pancreatitis?

A. Elevated blood urea nitrogen (BUN)


B. Elevated serum lipase
C. Elevated aspartate aminotransferase (AST)
D. Increased lactate dehydrogenase (LD)

21. A male client with cholelithiasis has a gallstone lodged in the


common bile duct. When assessing this client, the nurse expects to
note:

A. yellow sclera.
B. light amber urine.
C. circumoral pallor.
D. black, tarry stools.

22. Nurse Hannah is teaching a group of middle-aged men about peptic


ulcers. When discussing risk factors for peptic ulcers, the nurse should
mention:

A. a sedentary lifestyle and smoking.


B. a history of hemorrhoids and smoking.
C. alcohol abuse and a history of acute renal failure.
D. alcohol abuse and smoking.

23. While palpating a female clients right upper quadrant (RUQ), the
nurse would expect to find which of the following structures?

A. Sigmoid colon
B. Appendix
C. Spleen
D. Liver

24. A male client has undergone a colon resection. While turning him,
wound dehiscence with evisceration occurs. The nurses first response
is to:

A. call the physician.


B. place saline-soaked sterile dressings on the wound.
C. take a blood pressure and pulse.
D. pull the dehiscence closed.

25. The nurse is monitoring a female client receiving paregoric to


treat diarrhea for drug interactions. Which drugs can produce
additive constipation when given with an opium preparation?

A. Antiarrhythmic drugs
B. Anticholinergic drugs
C. Anticoagulant drugs
D. Antihypertensive drugs

26. A male client is recovering from an ileostomy that was performed to


treat inflammatory bowel disease. During discharge teaching, the nurse
should stress the importance of:

A. increasing fluid intake to prevent dehydration.


B. wearing an appliance pouch only at bedtime.
C. consuming a low-protein, high-fiber diet.
D. taking only enteric-coated medications.

27. The nurse is caring for a female client with active upper
GI bleeding. What is the appropriate diet for this client during the first
24 hours after admission?

A. Regular diet
B. Skim milk
C. Nothing by mouth
D. Clear liquids

28. A male client has just been diagnosed with hepatitis A. On


assessment, the nurse expects to note:

A. severe abdominal pain radiating to the shoulder.


B. anorexia, nausea, and vomiting.
C. eructation and constipation.
D. abdominal ascites.

29. A female client with viral hepatitis A is being treated in an acute


care facility. Because the client requires enteric precautions, the nurse
should:

A. place the client in a private room.


B. wear a mask when handling the clients bedpan.
C. wash the hands after touching the client.
D. wear a gown when providing personal care for the client.

30. Which of the following factors can cause hepatitis A?

A. Contact with infected blood


B. Blood transfusions with infected blood
C. Eating contaminated shellfish
D. Sexual contact with an infected person

Answers and Rationale

1. Answer: D. vitamin K

Intestinal bacteria synthesize such nutritional substances as vitamin K,


thiamine, riboflavin, vitamin B12, folic acid, biotin, and nicotinic acid.
Therefore, antibiotic therapy may interfere with synthesis of these substances,
including vitamin K. Intestinal bacteria dont synthesize vitamins A, D, or E.

2. Answer: B. decreased urine output.

Acute pancreatitis can cause decreased urine output, which results from the
renal failure that sometimes accompanies this condition. Intracranial pressure
neither increases nor decreases in a client with pancreatitis. Tachycardia, not
bradycardia, usually is associated with pulmonary or hypovolemic complications
of pancreatitis. Hypotension can be caused by a hypovolemic complication,
but hypertension usually isnt related to acute pancreatitis.

3. Answer: B. Lying on the left side with knees bent

For a colonoscopy, the nurse initially should position the client on the left side
with knees bent. Placing the client on the right side with legs
straight, prone with the torso elevated, or bent over with hands touching the
floor wouldnt allow proper visualization of the large intestine.
4. Answer: A. Tell me about your husbands alcohol usage.

A Mallory-Weiss tear is associated with massive bleeding after a tear occurs in


the mucous membrane at the junction of the esophagus and stomach. There is
a strong relationship between ethanol usage, resultant vomiting, and a Mallory-
Weiss tear. The bleeding is coming from the stomach, not from the lungs as
would be true in some cases of tuberculosis. A Mallory-Weiss tear doesnt occur
from chest injuries or falls and isnt associated with eating spicy foods.

5. Answer: A. Change the tube feeding solutions and tubing at least


every 24 hours.

Tube feeding solutions and tubing should be changed every 24 hours, or more
frequently if the feeding requires it. Doing so prevents contamination and
bacterial growth. The head of the bed should be elevated 30 to 45 degrees
continuously to prevent aspiration. Checking for gastrostomy tube placement is
performed before initiating the feedings and every 4 hours during continuous
feedings. Clients may ambulate during feedings.

6. Answer: B. 15 to 30 minutes

Meperidines onset of action is 15 to 30 minutes. It peaks between 30 and 60


minutes and has a duration of action of 2 to 4 hours.

7. Answer: C. Purpura and petechiae

A hepatic disorder, such as cirrhosis, may disrupt the livers normal use of
vitamin K to produce prothrombin (a clotting factor). Consequently, the nurse
should monitor the client for signs of bleeding, including purpura and petechiae.
Dyspnea and fatigue suggest anemia. Ascites and orthopnea are unrelated to
vitamin K absorption. Gynecomastia and testicular atrophy result from
decreased estrogen metabolism by the diseased liver.

8. Answer: B. Pancreatitis
Hypovolemic shock from fluid shifts is a major factor in acute pancreatitis. The
other conditions are less likely to exhibit fluid volume deficit.

9. Answer: A. Irrigate the tube with cola.

The nurse should irrigate the tube with cola because its effervescence and
acidity are suited to the purpose, its inexpensive, and its readily available in
most homes. Advancing the NG tube is inappropriate because the tube is
designed to stay in the stomach and isnt long enough to reach the intestines.
Applying intermittent suction or using a syringe for aspiration is unlikely to
dislodge the material clogging the tube but may create excess pressure.
Intermittent suction may even collapse the tube.

10. Answer: B. morphine may cause spasms of Oddis sphincter.

For a client with pancreatitis, the physician will probably avoid


prescribing morphine because this drug may trigger spasms of the sphincter of
Oddi (a sphincter at the end of the pancreatic duct), causing irritation of the
pancreas. Meperidine has a somewhat shorter duration of action than morphine.
The two drugs are equally addictive. Morphine isnt associated with hepatic
dysfunction.

11. Answer: C. Chronic low self-esteem

Young women with Chronic low self-esteem are at highest risk for anorexia
nervosa because they perceive being thin as a way to improve their self-
confidence. Hopelessness and Powerlessness are inappropriate nursing
diagnoses because clients with anorexia nervosa seldom feel hopeless or
powerless; instead, they use food to control their desire to be thin and hope
that restricting food intake will achieve this goal. Anorexia nervosa doesnt
result from a knowledge deficit, such as one regarding good nutrition.

12. Answer: A. Endoscopy


Endoscopy permits direct evaluation of the upper GI tract and can detect 90%
of bleeding lesions. An upper GI series, or barium study, usually isnt the
diagnostic method of choice, especially in a client with acute
active bleeding whos vomiting and unstable. An upper GI series is also less
accurate than endoscopy. Although an upper GI series might confirm the
presence of a lesion, it wouldnt necessarily reveal whether the lesion
is bleeding. Hb levels and HCT, which indicate loss of blood volume, arent
always reliable indicators of GI bleeding because a decrease in these values
may not be seen for several hours. Arteriography is an invasive study
associated with life-threatening complications and wouldnt be used for an initial
evaluation.

13. Answer: A. You may have eaten contaminated restaurant food.

Hepatitis A virus typically is transmitted by the oral-fecal route commonly by


consuming food contaminated by infected food handlers. The virus isnt
transmitted by the I.V. route, blood transfusions, or unprotected sex. Hepatitis
B can be transmitted by I.V. drug use or blood transfusion. Hepatitis C can be
transmitted by unprotected sex.

14. Answer: B. Obstruction of the appendix reduces arterial flow,


leading to ischemia, inflammation, and rupture of the appendix.

A client with appendicitis is at risk for infection related to inflammation,


perforation, and surgery because obstruction of the appendix causes mucus
fluid to build up, increasing pressure in the appendix and compressing venous
outflow drainage. The pressure continues to rise with venous obstruction;
arterial blood flow then decreases, leading to ischemia from lack of perfusion.
Inflammation and bacterial growth follow, and swelling continues to raise
pressure within the appendix, resulting in gangrene and rupture. Geriatric, not
middle-aged, clients are especially susceptible to appendix rupture.

15. Answer: D. cryoprecipitate and fresh frozen plasma.


The liver is vital in the synthesis of clotting factors, so when its diseased or
dysfunctional, as in hepatitis C, bleeding occurs. Treatment consists of
administering blood products that aid clotting. These include fresh frozen
plasma containing fibrinogen and cryoprecipitate, which have most of the
clotting factors. Although administering whole blood, albumin, and packed cells
will contribute to hemostasis, those products arent specifically used to treat
hemostasis. Platelets are helpful, but the best answer is cryoprecipitate and
fresh frozen plasma.

16. Answer: B. Avoid coffee and alcoholic beverages.

To prevent reflux of stomach acid into the esophagus, the nurse should advise
the client to avoid foods and beverages that increase stomach acid, such as
coffee and alcohol. The nurse also should teach the client to avoid lying down
after meals, which can aggravate reflux, and to take antacids after eating. The
client need not limit fluid intake with meals as long as the fluids arent gastric
irritants.

17. Answer: D. Administering I.V. fluids

I.V. infusions containing normal saline solution and potassium should be given
first to maintain fluid and electrolyte balance. For the clients comfort and to
assist in bowel decompression, the nurse should prepare to insert an NG
tube next. A blood sample is then obtained for laboratory studies to aid in the
diagnosis of bowel obstruction and guide treatment. Blood studies usually
include a complete blood count, serum electrolyte levels, and blood urea
nitrogen level. Pain medication often is withheld until obstruction is diagnosed
because analgesics can decrease intestinal motility.

18. Answer: B. The client is free from esophagitis and achalasia.

Dysphagia may be the reason why a client with esophagitis or achalasia seeks
treatment. Dysphagia isnt associated with rectal tenesmus, duodenal
inflammation, or abnormal gastric structures.
19. Answer: A. Notify the physician

An NG tube that fails to drain during the postoperative period should be


reported to the physician immediately. It may be clogged, which could increase
pressure on the suture site because fluid isnt draining adequately. Repositioning
or irrigating an NG tube in a client who has undergone gastric surgery can
disrupt the anastomosis. Increasing the level of suction may cause trauma to GI
mucosa or the suture line.

20. Answer: B. Elevated serum lipase

Elevation of serum lipase is the most reliable indicator of pancreatitis because


this enzyme is produced solely by the pancreas. A clients BUN is typically
elevated in relation to renal dysfunction; the AST, in relation to liver
dysfunction; and LD, in relation to damaged cardiac muscle.

21. Answer: A. yellow sclera.

Yellow sclerae may be the first sign of jaundice, which occurs when the common
bile duct is obstructed. Urine normally is light amber. Circumoral pallor and
black, tarry stools dont occur in common bile duct obstruction; they are signs
of hypoxia and GI bleeding, respectively.

22. Answer: D. alcohol abuse and smoking.

Risk factors for peptic (gastric and duodenal) ulcers include alcohol abuse,
smoking, and stress. A sedentary lifestyle and a history of hemorrhoids arent
risk factors for peptic ulcers. Chronic renal failure, not acute renal failure, is
associated with duodenal ulcers.

23. Answer: D. Liver

The RUQ contains the liver, gallbladder, duodenum, head of the pancreas,
hepatic flexure of the colon, portions of the ascending and transverse colon, and
a portion of the right kidney. The sigmoid colon is located in the left lower
quadrant; the appendix, in the right lower quadrant; and the spleen, in the left
upper quadrant.

24. Answer: B. place saline-soaked sterile dressings on the wound.

The nurse should first place saline-soaked sterile dressings on the open wound
to prevent tissue drying and possible infection. Then the nurse should call the
physician and take the clients vital signs. The dehiscence needs to be surgically
closed, so the nurse should never try to close it.

25. Answer: B. Anticholinergic drugs

Paregoric has an additive effect of constipation when used with anticholinergic


drugs. Antiarrhythmics, anticoagulants, and antihypertensives arent known to
interact with paregoric.

26. Answer: A. increasing fluid intake to prevent dehydration.

Because stool forms in the large intestine, an ileostomy typically drains liquid
waste. To avoid fluid loss through ileostomy drainage, the nurse should instruct
the client to increase fluid intake. The nurse should teach the client to wear a
collection appliance at all times because ileostomy drainage is incontinent, to
avoid high-fiber foods because they may irritate the intestines, and to avoid
enteric-coated medications because the body cant absorb them after an
ileostomy

27. Answer: C. Nothing by mouth

Shock and bleeding must be controlled before oral intake, so the client should
receive nothing by mouth. A regular diet is incorrect. When the bleeding is
controlled, the diet is gradually increased, starting with ice chips and then clear
liquids. Skim milk shouldnt be given because it increases gastric acid
production, which could prolong bleeding. A liquid diet is the first diet offered
after bleeding and shock are controlled.
28. Answer: B. anorexia, nausea, and vomiting.

Hallmark signs and symptoms of hepatitis A include anorexia, nausea,


vomiting, fatigue, and weakness. Abdominal pain may occur but doesnt radiate
to the shoulder. Eructation and constipation are common in gallbladder disease,
not hepatitis A. Abdominal ascites is a sign of advanced hepatic disease, not an
early sign of hepatitis A.

29. Answer: C. wash the hands after touching the client.

To maintain enteric precautions, the nurse must wash the hands after touching
the client or potentially contaminated articles and before caring for another
client. A private room is warranted only if the client has poor hygiene for
instance, if the client is unlikely to wash the hands after touching infective
material or is likely to share contaminated articles with other clients. For enteric
precautions, the nurse need not wear a mask and must wear a gown only if
soiling from fecal matter is likely.

30. Answer: C. Eating contaminated shellfish

Hepatitis A can be caused by consuming contaminated water, milk, or food


especially shellfish from contaminated water. Hepatitis B is caused by blood and
sexual contact with an infected person. Hepatitis C is usually caused by contact
with infected blood, including receiving blood transfusions.

1. The correct sequence for abdominal assessment is:

A. inspection, percussion, palpation, auscultation.


B. inspection, auscultation, palpation, percussion.
C. inspection, palpation, auscultation, percussion.
D. inspection, percussion, auscultation, palpation.

2. Peritonitis can occur as a complication of:


A. septicemia
B. multiple organ failure
C. hypovolemic shock
D. peptic ulcer disease

3. A patient has become very depressed postoperatively after


receiving colostomy for GI cancer. He does not participate in his
colostomy care or look at the stoma. An appropriate nursing diagnosis
for this situation is:

A. Ineffective Individual Coping


B. Knowledge Deficit
C. Impaired Adjustment
D. Anxiety

4. Patients with esophageal varices would reveal the following


assessment:

A. increased blood pressure.


B. increased heart rate.
C. decreased respiratory rate.
D. increased urinary output.

5. The nurse would anticipate using which medication if sclerotherapy


has not been used?

A. neomycin
B. propranolol
C. vasopressin
D. cimetidine

6. The nurse must be alert for complications with Sengstaken-


Blakemore intubation including:
A. pulmonary obstruction.
B. pericardiectomy syndrome
C. pulmonary embolization
D. cor pulmonale

7. Peptic ulcer disease may be caused by which of the following?

A. Helicobacter pylori
B. Clostridium difficile
C. Candida albicans
D. Staphylococcus aureus

8. Pain control with peptic ulcer disease includes all of the following
except:

A. promoting physical and emotional rest.


B. identifying stressful situations.
C. eating meals when desired.
D. administering medications that decrease gastric acidity.

9. Nitrosocarcinogen production can be inhibited with intake of:

A. vitamin C.
B. vitamin E.
C. carbohydrates.
D. fiber.

10. The nurse can expect a 60-year old patient with ischemic bowel to
report a history of:

A. diabetes mellitus
B. asthma
C. Addisons Disease
D. cancer of the bowel

11. During initial assessment of a patient post-endoscopy, the nurse


notes absent bowel sounds, tachycardia, and abdominal distention. The
nurse would anticipate:

A. ischemic bowel
B. peritonitis
C. hypovolemic shock
D. perforated bowel

12. Which of the following tests can be useful as a diagnostic and


therapeutic tool in the biliary system?

A. Ultrasonography.
B. MRI
C. Endoscopic retrograde cholangiopancreatography (ERCP)
D. Computed tomography scan (CT scan)

13. To inhibit pancreatic secretions, which pharmacologic agent would


you anticipate administering to a patient with acute pancreatitis?

A. Nitroglycerin
B. Somatostatin
C. Pancrelipase
D. Pepcid

14. Your patients ABG reveal an acidic pH, an acidic CO2 and a normal
bicarbonate level. Which of the following indicates this acid-base
disturbance?
A. Respiratory acidosis
B. Respiratory alkalosis
C. Metabolic acidosis
D. Metabolic alkalosis

15. A clinical manifestation of acute pancreatitis is epigastric pain. Your


nursing intervention to facilitate relief of pain would place the patient
in a:

A. knee-chest position
B. semi-Fowlers position
C. recumbent position
D. low-Fowlers position

16. What assessment finding of a patient with acute pancreatitis would


indicate a bluish discoloration around the umbilicus?

A. Grey-Turners sign
B. Homans sign
C. Rovsings sign
D. Cullens sign

17. A patient with severe cirrhosis of the liver develops hepatorenal


syndrome. Which of the following nursing assessment data would
support this?

A. oliguria and azotemia


B. metabolic alkalosis
C. decreased urinary concentration
D. weight gain of less than 1 lb per week

18. Which phase of hepatitis would the nurse incur strict precautionary
measures at?
A. icteric
B. non-icteric
C. post-icteric
D. pre-icteric

19. You are caring for Rona, a 35-year-old female in a hepatic coma.
Which evaluation criteria would be the most appropriate?

A. The patient demonstrates an increase in level of consciousness.


B. The patient exhibits improved skin integrity.
C. The patient experiences no evident signs of bleeding.
D. The patient verbalize decreased episodes of pain.

20. What is the primary nursing diagnosis for a 4th to 10th day
postoperative liver transplant patient?

A. Excess Fluid Volume


B. Risk for Rejection
C. Impaired Skin Integrity
D. Decreased Cardiac Output

21. Which of the following laboratory values would be the most


important to monitor for a patient with pancreatic cancer?

A. Serum glucose
B. Radioimmunoassay (RIA)
C. Creatine phosphokinase (CPK)
D. Carcinoembryonic antigen (CEA)

22. You observe changes in mentation, irritability, restlessness, and


decreased concentration in a patient with cancer of the liver. Hepatic
encephalopathy is suspected and the patient is ordered neomycin
enemas. Which of the following information in the patients history
would be a contraindication of this order?

A. left nephrectomy
B. glaucoma in both eyes
C. myocardial infarction
D. peripheral neuropathy

23. A nursing intervention for a patient with hepatitis B would include


which of the following types of isolation.

A. Universal precautions
B. Blood transfusions
C. Enteric isolation
D. Strict isolation

24. A patient is admitted with lacerated liver as a result of blunt


abdominal trauma. Which of the following nursing interventions would
NOT be appropriate for this patient?

A. Monitor for respiratory distress.


B. Monitor for coagulation studies.
C. Administer pain medications as ordered.
D. Administer normal saline, crystalloids as ordered.

25. A male client is recovering from a small-bowel resection. To


relieve pain, the physician prescribes meperidine (Demerol), 75 mg I.M.
every 4 hours. How soon after administration should meperidine onset
of action occur?

A. 5 to 10 minutes
B. 15 to 30 minutes
C. 30 to 60 minutes
D. 2 to 4 hours

Answers and Rationale

1. Answer: B. inspection, auscultation, palpation, percussion

Auscultation is done before palpation to avoid stimulating peristaltic movements


and distorting auscultatory sounds.

2. Answer: D. peptic ulcer disease

Perforation is a life-threatening complication of peptic ulcer disease and can


result in peritonitis.

3. Answer: A. Ineffective Individual Coping

The patient is dealing with a disturbance in self-concept and difficulty coping


with the newly established stoma.

4. Answer: B. increased heart rate.

Tachycardia is an early sign of compensation for patients with esophageal


varices.

5. Answer: C. vasopressin

Vasopressin is the drug of choice when sclerotherapy is contraindicated.


Neomycin is used in preventing encephalopathy when blood is broken
down. Propranolol may or may not be used to decrease cardiac output and
hepatic venous pressure.
6. Answer: A. pulmonary obstruction.

Rupture or deflation of the balloon could result in upper airway obstruction. The
other choices are not related to the tube.

7. Answer: A. Helicobacter pylori

Helicobacter pylori is considered to be the major cause of ulcer formation. Other


choices are not related to ulcer formation.

8. Answer: C. eating meals when desired.

Meals should be regularly spaced in a relaxed environment. Choices A, B, and D


are proper interventions in providing pain control.

9. Answer: A. vitamin C.

Vitamin C and refrigeration of foods inhibit nitroso carcinogen.

10. Answer: A. diabetes mellitus

Ischemic bowel occurs in patient over 50 with a history of diabetes mellitus.

11. Answer: D. perforated bowel

Invasive diagnostic testing can cause perforated bowel. Ischemic bowel (Choice
A) is usually not related. Peritonitis (Choice B) can be a complication after initial
perforation. Hypovolemic shock (Choice C) can occur if peritonitis is allowed to
continue.

12. Answer: C. Endoscopic retrograde cholangiopancreatography


(ERCP)
ERCP permits direct visualization of the pancreatic and common bile ducts. Its
therapeutic value is in retrieving gallstones from the distal and common bile
ducts and dilating strictures. Ultrasonography (Option A) aids in the diagnosis
of cholecystitis, gallstones, pancreatitis, and metastatic disease. It also
identifies edema, inflammation, and fatty or fibrotic infiltrates or calcifications.
MRI (Option B) detects hepatic neoplasms, cysts, abscesses, and hematomas. A
CT Scan (Option D) can be done with our without a contrast medium. It can
detect tumors, cysts, pseudocysts, abscesses, hematomas, and obstructions of
the liver, biliary tract and pancreas.

13. Answer: B. Somatostatin

Somatostatin, a treatment for acute pancreatitis, inhibits the release of


pancreatic enzymes. Nitroglycerin (Option A) is a vasodilator and does not
affect pancreatic secretions. Pancrelipase (Option C) is an enzyme that aids in
the digestion and absorption of fats and proteins. Pepcid is an H2 blocker and is
used to decrease gastric motility.

14. Answer: A. Respiratory acidosis

A pH of 7.35 indicates acidosis, as does an acidic CO2 and bicarbonate. See


also: 8-Step Guide to ABG Analysis: Tic-Tac-Toe Method

15. Answer: A. knee-chest position

Flexion of the trunk lessens the pain and decreases restlessness. Other
positions do not decrease the pain.

16. Answer: D. Cullens sign

Cullens sign is associated with pancreatitis when a hemorrhage is suspected.


Grey-Turners sign is ecchymosis in the flank area suggesting retroperitoneal
bleed. Homans sign is cal pain elicited by the dorsiflexion of the foot and
suggests deep vein thrombosis. Rovsings sign is associated
with appendicitis when pain is felt with pressure at McBurneys point.

17. Answer: A. oliguria and azotemia

Hepatorenal syndrome is a functional disorder resulting from a


redistribution of renal blood flow. Oliguria and azotemia occur abruptly
as a result of this complication.

For Option B, excess organic acids are not being excreted by the
damaged kidneys, resulting in an elevated concentration of hydrogen
ions; decreased pH occurs, causing metabolic acidosis.

For Option C, concentration of the urine is increased with decreased


renal function.

For Option D, with renal insufficiency, significant weight gain is


expected due to fluid retention.

18. Answer: D. Pre-icteric

Pre-icteric is the infective phase and precautionary measures should be strictly


enforced. However, most patients are not always diagnosed during this phase.
For Option A and C, precautionary measures should already be in placed. For
Option B, there is no non-icteric phase.

19. Answer: A. The patient demonstrates an increase in level of


consciousness.

Increased level of consciousness indicates resolving of a comatose state. Other


options are important evaluation but do not evaluate a patient in a hepatic
coma who is responding to external stimuli.

20. Answer: B. Risk for Rejection


Risk for rejection is always a possibility, especially during the 4th to 10th day
postoperatively.

21. Answer: A. Serum glucose

In pancreatitis, hypersecretion of the insulin from a tumor may affect the islets
of Langerhans, resulting in hyperinsulinemia, a complication of
pancreatic cancer. Options B and D, should also be monitored to measure the
effects of therapy, but hypoglycemia may be life-threatening. Creatine
phosphokinase is an enzyme that reflects normal tissue catabolism. Elevated
serum levels indicate trauma to cells with high CPK content. CPK and CPK-
isoenzymes are used to detect a myocardial infarction.

22. Answer: A. left nephrectomy

Neomycin prevents the release of ammonia from the intestinal bacteria flora
and from the breakdown of red blood cells. Common side-effects of this drug
are nephrotoxicity and ototoxicity. Patients with renal disease or renal
impairment should not take this drug. Peripheral neuropathy (Option D) is a
chronic complication of diabetes mellitus. Options B and C are not affected by
neomycin.

23. Answer: A. Universal precautions

Universal precautions are indicated for the patient with hepatitis B. Hepatitis
B is contracted via blood and blood products, body secretions, and punctures
from contaminated needles.

24. Answer: C. Administer pain medications as ordered.

Pain medication may mask signs and symptoms of hemorrhage, further


decrease blood pressure, and interfere with assessment of neurologic status and
additional abdominal injury.
25. Answer B. 15 to 30 minutes

Meperidine onset of action is 15 to 30 minutes. It peaks between 30 and 60


minutes and has a duration of action of 2 to 4 hours.

1. For a client in hepatic coma, which outcome would be the most


appropriate?

A. The client is oriented to time, place, and person.


B. The client exhibits no ecchymotic areas.
C. The client increases oral intake to 2,000 calories/day.
D. The client exhibits increased serum albumin level.

2. Jordin is a client with jaundice who is experiencing pruritus. Which


nursing intervention would be included in the care plan for the client?

A. Administering vitamin K subcutaneously


B. Applying pressure when giving I.M. injections
C. Decreasing the clients dietary protein intake
D. Keeping the clients fingernails short and smooth

3. Marie, a 51-year-old woman, is diagnosed with cholecystitis. Which


diet, when selected by the client, indicates that the nurses teaching
has been successful?

A. 4-6 small meals of low-carbohydrate foods daily


B. High-fat, high-carbohydrate meals
C. Low-fat, high-carbohydrate meals
D. High-fat, low protein meals

4. The hospital administrator had undergone percutaneous transhepatic


cholangiography. which assessment finding indicates complication after
the operation?
A. Fever and chills
B. Hypertension
C. Bradycardia
D. Nausea and diarrhea

5. When planning home care for a client with hepatitis A, which


preventive measure should be emphasized to protect the clients
family?

A. Keeping the client in complete isolation


B. Using good sanitation with dishes and shared bathrooms
C. Avoiding contact with blood-soiled clothing or dressing
D. Forbidding the sharing of needles or syringes

6. For Jayvin who is taking antacids, which instruction would be


included in the teaching plan?

A. Take the antacids with 8 oz of water.


B. Avoid taking other medications within 2 hours of this one.
C. Continue taking antacids even when pain subsides.
D. Weigh yourself daily when taking this medication.

7. Which clinical manifestation would the nurse expect a client


diagnosed with acute cholecystitis to exhibit?

A. Jaundice, dark urine, and steatorrhea


B. Acute right lower quadrant (RLQ) pain, diarrhea, and dehydration
C. Ecchymosis petechiae, and coffee-ground emesis
D. Nausea, vomiting, and anorexia

8. Pierre who is diagnosed with acute pancreatitis is under the care of


Nurse Bryan. Which intervention should the nurse include in the care
plan for the client?
A. Administration of vasopressin and insertion of a balloon tamponade
B. Preparation for a paracentesis and administration of diuretics
C. Maintenance of nothing-by-mouth status and insertion of nasogastric (NG)
tube with low intermittent suction
D. Dietary plan of a low-fat diet and increased fluid intake to 2,000 ml/day

9. When teaching a client about pancreatic function, the nurse


understands that pancreatic lipase performs which function?

A. Transports fatty acids into the brush border


B. Breaks down fat into fatty acids and glycerol
C. Triggers cholecystokinin to contract the gallbladder
D. Breaks down protein into dipeptides and amino acids

10. A 52-year-old man was referred to the clinic due to increased


abdominal girth. He is diagnosed with ascites by the presence of a fluid
thrill and shifting dullness on percussion. After administering diuretic
therapy, which nursing action would be most effective in ensuring safe
care?

A. Measuring serum potassium for hyperkalemia


B. Assessing the client for hypervolemia
C. Measuring the clients weight weekly
D. Documenting precise intake and output

11. Which assessment finding indicates that lactulose is effective in


decreasing the ammonia level in the client with hepatic
encephalopathy?

A. Passage of two or three soft stools daily


B. Evidence of watery diarrhea
C. Daily deterioration in the clients handwriting
D. Appearance of frothy, foul-smelling stools
12. Nurse Farrah is providing care for Kristoff who has jaundice. Which
statement indicates that the nurse understands the rationale for
instituting skin care measures for the client?

A. Jaundice is associated with pressure ulcer formation.


B. Jaundice impairs urea production, which produces pruritus.
C. Jaundice produces pruritus due to impaired bile acid excretion.
D. Jaundice leads to decreased tissue perfusion and subsequent breakdown.

13. Which rationale supports explaining the placement of an


esophageal tamponade tube in a client who is hemorrhaging?

A. Allowing the client to help insert the tube


B. Beginning teaching for home care
C. Maintaining the clients level of anxiety and alertness
D. Obtaining cooperation and reducing fear

14. For Rico who has chronic pancreatitis, which nursing intervention
would be most helpful?

A. Allowing liberalized fluid intake


B. Counseling to stop alcohol consumption
C. Encouraging daily exercise
D. Modifying dietary protein

15. Mr. Hasakusa is in end-stage liver failure. Which interventions


should the nurse implement when addressing hepatic encephalopathy?
(Select all that apply.)

A. Assessing the clients neurologic status every 2 hours


B. Monitoring the clients hemoglobin and hematocrit levels
C. Evaluating the clients serum ammonia level
D. Monitoring the clients handwriting daily
E. Preparing to insert an esophageal tamponade tube
F. Making sure the clients fingernails are short

16. For a client with hepatic cirrhosis who has altered clotting
mechanisms, which intervention would be most important?

A. Allowing complete independence of mobility


B. Applying pressure to injection sites
C. Administering antibiotics as prescribed
D. Increasing nutritional intake

17. A client with advanced cirrhosis has been diagnosed with hepatic
encephalopathy. The nurse expects to assess for:

A. Malaise
B. Stomatitis
C. Hand tremors
D. Weight loss

18. A client diagnosed with chronic cirrhosis who has ascites and
pitting peripheral edema also has hepatic encephalopathy. Which of the
following nursing interventions are appropriate to prevent skin
breakdown? (Select all that apply.)

A. Range of motion every 4 hours


B. Turn and reposition every 2 hours
C. Abdominal and foot massages every 2 hours
D. Alternating air pressure mattress
E. Sit in chair for 30 minutes each shift

19. Which of the following will the nurse include in the care plan for a
client hospitalized with viral hepatitis?
A. Increase fluid intake to 3000 ml per day
B. Adequate bed rest
C. Bland diet
D. Administer antibiotics as ordered

20. Spironolactone (Aldactone) is prescribed for a client with


chronic cirrhosis and ascites. The nurse should monitor the client for
which of the following medication-related side effects?

A. Jaundice
B. Hyperkalemia
C. Tachycardia
D. Constipation

Answers and Rationale

Here are the answers for this exam. Gauge your performance by counter
checking your answers to those below. If you have any disputes or clarifications,
please direct them to the comments section.

1. Answer: A. The client is oriented to time, place, and person.

Hepatic coma is the most advanced stage of hepatic encephalopathy. As hepatic


coma resolves, improvement in the clients level of consciousness occurs. The
client should be able to express orientation to time, place, and person.
Ecchymotic areas are related to decreased synthesis of clotting factors.
Although oral intake may be related to level of consciousness, it is more closely
related to anorexia. The serum albumin level reflects hepatic synthetic ability,
not level of consciousness.

2. Answer: D. Keeping the clients fingernails short and smooth


The client with pruritus experiences itching, which may lead to skin breakdown
and possibly infection from scratching. Keeping his fingernails short and smooth
helps prevent skin breakdown and infection from scratching. Applying pressure
when giving I.M. injections and administering vitamin K subcutaneously are
important if the client develops bleeding problems. Decreasing the clients
dietary intake is appropriate if the clients ammonia levels are increased.

3. Answer: C. Low-fat, high-carbohydrate meals

For the client with cholecystitis, fat intake should be reduced. The calories from
fat should be substituted with carbohydrates. Reducing carbohydrate intake
would be contraindicated. Any diet high in fat may lead to another attack of
cholecystitis.

4. Answer: A. Fever and chills

Septicemia is a common complication after a percutaneous transhepatic


cholangiography. Evidence of fever and chills, possibly indicative of septicemia,
is important. Hypotension, not hypertension, is associated with septicemia.
Tachycardia, not bradycardia, is most likely to occur. Nausea and diarrhea may
occur but are not classic signs of sepsis.

5. Answer: B. Using good sanitation with dishes and shared bathrooms

Hepatitis A is transmitted through the fecal oral route or from contaminated


water or food. Measures to protect the family include good handwashing,
personal hygiene and sanitation, and use of standard precautions. Complete
isolation is not required. Avoiding contact with blood-soiled clothing or dressings
or avoiding the sharing of needles or syringes are precautions needed to
prevent transmission of hepatitis B.

6. Answer: B. Avoid taking other medications within 2 hours of this


one.
Antacids neutralize gastric acid and decrease the absorption of other
medications. The client should be instructed to avoid taking other medications
within 2 hours of the antacid. Water, which dilutes the antacid, should not be
taken with antacid. A histamine receptor antagonist should be taken even
when pain subsides. Daily weights are indicated if the client is taking a diuretic,
not an antacid.

7. Answer: D. Nausea, vomiting, and anorexia

Acute cholecystitis is an acute inflammation of the gallbladder commonly


manifested by the following: anorexia, nausea, and vomiting; biliary colic;
tenderness and rigidity the right upper quadrant (RUQ) elicited on palpation
(e.g., Murphys sign); fever; fat intolerance; and signs and symptoms of
jaundice. Ecchymosis, petechiae, and coffee-ground emesis are clinical
manifestations of esophageal bleeding. The coffee-ground appearance indicates
old bleeding. Jaundice, dark urine, and steatorrhea are clinical manifestations of
the icteric phase of hepatitis.

8. Answer: C. Maintenance of nothing-by-mouth status and insertion of


nasogastric (NG) tube with low intermittent suction

With acute pancreatitis, the client is kept on nothing-by-mouth status to inhibit


pancreatic stimulation and secretion of pancreatic enzymes. NG intubation with
low intermittent suction is used to relieve nausea and vomiting, decrease painful
abdominal distention, and remove hydrochloric acid. Vasopressin would be
appropriate for a client diagnosed with bleeding esophageal varices.
Paracentesis and diuretics would be appropriate for a client diagnosed with
portal hypertension and ascites. A low-fat diet and increased fluid intake would
further aggravate the pancreatitis.

9. Answer: B. Breaks down fat into fatty acids and glycerol


Lipase hydrolyses or breaks down fat into fatty acids and glycerol. Lipase is not
involved with the transport of fatty acids into the brush border. Fat itself
triggers cholecystokinin release. Protein breakdown into dipeptides and amino
acids is the function of trypsin, not lipase.

10. Answer: D. Documenting precise intake and output

For the client with ascites receiving diuretic therapy, careful intake and output
measurement is essential for safe diuretic therapy. Diuretics lead to fluid losses,
which if not monitored closely and documented, could place the client at risk for
serious fluid and electrolyte imbalances. Hypokalemia, not hyperkalemia,
commonly occurs with diuretic therapy. Because urine output increases, a client
should be assessed for hypovolemia, not hypervolemia. Weights are also an
accurate indicator of fluid balance. However, for this client, weights should be
obtained daily, not weekly.

11. Answer: A. Passage of two or three soft stools daily

Lactulose reduces serum ammonia levels by inducing catharsis, subsequently


decreasing colonic pH and inhibiting fecal flora from producing ammonia from
urea. Ammonia is removed with the stool. Two or three soft stools daily indicate
effectiveness of the drug. Watery diarrhea indicates overdose. Daily
deterioration in the clients handwriting indicates an increase in the ammonia
level and worsening of hepatic encephalopathy. Frothy, foul-smelling stools
indicate steatorrhea, caused by impaired fat digestion.

12. Answer: C. Jaundice produces pruritus due to impaired bile acid


excretion.

Jaundice is a symptom characterized by increased bilirubin concentration in the


blood. Bile acid excretion is impaired, increasing the bile acids in the skin and
causing pruritus. Jaundice is not associated with pressure ulcer formation.
However, edema and hypoalbuminemia are. Jaundice itself does not impair urea
production or lead to decreased tissue perfusion.

13. Answer: D. Obtaining cooperation and reducing fear

An esophageal tamponade tube would be inserted in critical situations. Typically,


the client is fearful and highly anxious. The nurse therefore explains about the
placement to help obtain the clients cooperation and reduce his fear. This type
of tube is used only short term and is not indicated for home use. The tube is
large and uncomfortable. The client would not be helping to insert the tube. A
clients anxiety should be decreased, not maintained, and depending on the
degree of hemorrhage, the client may not be alert.

14. Answer: B. Counseling to stop alcohol consumption

Chronic pancreatitis typically results from repeated episodes of


acute pancreatitis. More than half of chronic pancreatitis cases are associated
with alcoholism. Counseling to stop alcohol consumption would be the most
helpful for the client. Dietary protein modification is not necessary for
chronic pancreatitis. Daily exercise and liberalizing fluid intake would be helpful
but not the most beneficial intervention.

15. Answer: A, C, D

Hepatic encephalopathy results from an increased ammonia level due to the


livers inability to covert ammonia to urea, which leads to neurologic dysfunction
and possible brain damage. The nurse should monitor the clients neurologic
status, serum ammonia level, and handwriting. Monitoring the clients
hemoglobin and hematocrit levels and insertion of an esophageal tamponade
tube address esophageal bleeding. Keeping fingernails short address jaundice.

16. Answer: B. Applying pressure to injection sites


The client with cirrhosis who has altered clotting is at high risk for hemorrhage.
Prolonged application of pressure to injection or bleeding sites is important.
Complete independence may increase the clients potential for injury, because
an unsupervised client may injure himself and bleed excessively. Antibiotics and
good nutrition are important to promote liver regeneration. However, they are
not most important for a client at high risk for hemorrhage.

17. Answer: C. Hand tremors

Hepatic encephalopathy results from the accumulation of neurotoxins in the


blood, therefore the nurse wants to assess for signs of neurological
involvement. Flapping of the hands (asterixis), changes in mentation, agitation,
and confusion are common. These clients typically have ascites and edema so
experience weight gain. Malaise and stomatitis are not related to neurological
involvement.

18. Answer: B, D

Edematous tissue must receive meticulous care to prevent tissue breakdown.


Range of motion exercises preserve joint function but do not prevent skin
breakdown. Abdominal or foot massage will not prevent skin breakdown but
must be cleansed carefully to prevent breaks in skin integrity. The feet should
be kept at the level of heart or higher so Fowlers position should be employed.
An air pressure mattress, careful repositioning can prevent skin breakdown.

19. Answer: B. Adequate bed rest

Treatment of hepatitis consists of bed rest during the acute phase to reduce
metabolic demands on the liver, thus increasing blood supply and cell
regeneration. Forcing fluids, antibiotics, and bland diets are not part of the
treatment plan for viral hepatitis.

20. Answer: B. Hyperkalemia


This is a potassium-sparing diuretic so clients should be monitored closely
for hyperkalemia. Diarrhea, dizziness, and headaches are other more common
side effects. Tachycardia, jaundice, and constipation are not expected side
effects of spironolactone (Aldactone).

You might also like